You are on page 1of 60

1.

2.
3.
4.
5.
6.
7.
8.
9.
10.
11.
12.
13.
14.
15.
16.
17.
18.
19.
20.
21.
22.
23.
24.
25.
26.
27.
28.
INTRODUCTION
IMPORTANTTERMSANDDEFINITIONS
INSTANTANEOUS RATEOFTHEREACTION
FACTORSAFFECTINGTHERATEOFREACTION
SPECIFICREACTIONRATE
2
2
4
4
5
DISTINCTIONBETWEENRATEOFREACTIONANDRATECONSTANT
RATELAW
ORDEROFREACTION
MOLECULARITYOFAREACTION
ELEMENTARYREACTIONS
COMPLEXREACTIONS
EQUILIBRIUMAPPROACH
STEADYSTATEAPPROXIMATION
TRUEANDAPPARENT RATECONSTANT
PSEUDOUNIMOLECULARREACTION
TYPES OFREACTIONS BASEDONORDEROFREACTION
MISCELLENEOUSREACTIONS
DETERMINATIONOFORDEROFREACTION
TEMPERATUREEFFECT
TEMPERATURE DEPENDENCEOFRATECONSTANTS
MECHANISMBASEDON INTERMEDIATEFORMATION
ACTIVATIONENERGYANDTHERATEOFREACTION
ACTIVATIONENERGIESOFAREVERSIBLE REACTION
ACTIVATIONENERGIESANDENERGYCHANGEDURINGREACTION
MAXWELL'S- BOLTZMANN DISTRIBUTIONCURVE
EFFECTOFCATALYST
COLLISIONTHEORYOFCHEMICALREACTIONS
5
6
6
7
7
7
7
8
9
9
10
15
19
20
20
24
24
25
26
26
27
27
THRESHOLDENERGYANDORIENTATIONOFCOLLIDINGMOLECULES
1
28
CONTENTS
S.NO. TOPIC PAGE NO
Chemical Kinetics
1. INTRODUCTION :
Chemical Kinetics means study of rate of chemical reactions. Rate of reaction depends onamount of
change inconcentration of reactants per unit time. Rate of reaction is influenced byvarious factors like
nature of substance, physical state of substance, temperature, concentration, presence of catalyst, etc.
Classificationof reactions : [In terms of rates] :
(i) Fast reactions too fast e.g. Detonation of explosives, acid-base neutraliztion, precipitation of
AgCl byNaCl andAgNO .
3
(ii) Moderate reaction Neither too fast nor too slowe.g. combination of H and Cl in presence of
2
light, hydroysis of ethyl acetate catalyzed by acid, decomposition of azomethane.
2
(iii) VerySlowreactionThere are certain reactions whichare too slowe.g. rusting ofiron, weathering
of rocks.
IMPORTANT TERMS AND DEFINITIONS :
2.1 RATEOFREACTION
It is defined as the change inconcentration of reactant (or product) in a particular time interval. Unit
of rate of reaction is mol L
1
s
1
.
Change in the concentration of reactants or (Products)
2.
Rate of reaction =
Time
2.2AVERAGERATEOFREACTION
Change in the concentration of reactants or products measured over bigger interval of time is called
average reaction rate. If Ac is the change inthe concentration of reactants and product inAt time, then
Change in the concentration of reactan ts or (Products)
Average Rate =
Time
C
2
C
1

AC
= =
t t
At
2 1
Unit of concentration gram mole / Litre
Unit of average velocity = =
Unit of time Second
= grammole litre
1
second
1
n
1
A+ n
2
B m
1
C+ m
2
D Note: For the reaction
1 A[A] A[B] A[C] A[D] 1 1 1
Rate of reaction = = = + = +
At At At At n
1
n
2
m
1
m
2
2
CHEMICALKINETICS
Chemical Kinetics
Note: For the reaction N
2
+ 3H
2
2NH
3
A[NH
3
]
(i) Rate of formation of ammonia = +
At
A[N
2
]
(ii) Rate of disappearance of nitrogen =
At
A[H
2
]
(iii) Rate of disappearance of hydrogen =
At
1 A[NH
3
] A[N
2
] A[H
2
] 1
Rate = + = =
2 At At
3
At
A[N
2
]
=
1 A[NH
3
]
Thus, Rate =
At 2 At
or rate of formation of ammonia =Twice the rate of disappearance of nitrogen

A[H
2
] (
A[NH
3
] 2
i.e. =
(
At
At 3
Ex.1 For the reaction RP, the concentration of a reactant changes from0.03 Mto 0.02Min 25 minutes.
Calculate the average rate of reaction using units of time bothinminutes andseconds.
A[R]
Average rate =
[R] [R]
=
2 1
Sol.
At t
2
t
1
=
0.02M 0.030M 0.01M
=
25minutes 25minutes
= 4 10
4
mol litre
1
minutes
1
0.01M
= = 6.66 10
6
mol litre
1
second
1
.
2560s
Ex.2 In a reaction, 2AProducts, the concentration ofAdecreases form0.5 mol L
1
to 0.04 mol L
1
in 10
minutes. Calculate the rate during this interval.
Average rate =
1 A[A]
=
1 [A]
2
[A]
1
Sol.
2 t
2
t
1 2 At
0.4M 0.5M
=
1

=
1 0.1M

10minutes 2 2 10 minutes
= 5 10
3
mole minutes
1
.
3
Chemical Kinetics
3. INSTANTANEOUS RATE OF THE REACTION
It is the rate of reaction whenthe average rate is taken over a
veryshort interval of time or rate of reaction at a particular time
is knownas instantaneous rate.
:
Lim
Ac dc
Instantaneous rate = =
dt At 0 At
Note: For the reaction n
1
A+ n
2
B m
1
C + m
2
D
1 d[A] 1 d[B] 1 d[C] 1 d[D]
Instantaneous Rate of reaction = = = + = +
n
1
dT n
2
dT m
1
dT m
2
dT
Ex.3
Sol.
For the rate of reaction : N
2
+ 3H
2
2NH
3
in terms of the concentrations of N
2
, H
2
and NH
3
can be expressed
as
d[N
2
]
=
1
=
d[H
2
] 1 d[ NH
3
]
Rate = +
2 dt 3 dt dt
Ex.4 What should be (a) the rate of disappearance of B and (b) the rate of formation of C, if the rate of
disappearance ofAfor the reactionA+ B2Cis 10
2
mole/litre/second at a particular temperature ?
(a) Rate of disappearance ofA= Rate of disappearance of B= 10
2
mole/litre/second
1
Rate of disappearance of A= Rate of formation of C
Sol.
2
(b) Rate of formation of C = 2 Rate of disappearance of A= 2 10
2
mole/litre/second
4. FACTORS AFFECTING THE RATE OF REACTION :
(i) Concentration : Lawof mass action enunciates that greater is the conc. of the reactants, the more
rapidly the reaction proceeds.
(ii) Pressure (Gaseous reaction) : Onincreasing the pressure, volume decreases andconc. increases
hence the rate of reaction in creases.
(iii) Temperature : It is generallyobserved that rise intemperature increases the reaction rate. It has
been found that rate is either doubled or tripled for every10 rise intemperature. Temp. coefficient
of reaction rate
k
T
+10
~ 2 or 3
k
T
Where
T + 10
and k
T
are rate constants at two temp. differing by 10.
(iv) Nature of the reactants : The rate depends upon specific bonds involved and hence onthe nature
of reactants.
(v) Surface area of the reactants : In heterogeneous reactions, more powered is the formof reactants,
more is the velocity. [as more active centres are provided]
(vi) Catalyst : Affects the rate immensely. Presence of positive catalyst increases the rate of reaction by
decreasing the activationenergy and presence of negative catalyst decreases the rate of reaction by
increasing the activation enrgy. Acatalyst mainlyaffects the activation energyof reaction and hence,
the rate constant and rate of reaction changes.
(vii) Intensity of Radiation: The rate of photochemical reactions normally increases with increase in
intensityof radiation.
(viii) pHof the medium
4
Chemical Kinetics
5. SPECIFIC REACTION RATE :
Applying lawof mass action to the reaction :
n
1
A+ n
2
B m
1
C + m
2
D
Rate [A]
n
1
[B]
n
2
or r = k [A]
n
1
[B]
n
2
This equation is known as rate law. Where k is the proportionality constant and is called Rate constant
or Rate coefficient (Specific reaction Rate)
On putting [A] = [B] = 1, we have : r = k
Hence, specific reaction rate is the rate of the reaction whenthe concentration of each reactant istaken
as unit.
Unit of Specific Reaction Rate Note:
conc.
r = k
[A]
n
1
[B]
n
2
= k
[conc.]
n
1
+n
2

time
[time]
1
k =
[conc.]
[1( n
1
+n
2
)]
[1-(n +n )]
mole (
1 2

litre
(
1
or k = .[second]

where n
1
+ n
2
= n which is order of reaction
| mole |
1n
1
K = second
|
litre
\ .
k = mol.lit
1
.sec
1
k = sec
1
k = mol
1
.lit
1
.sec
1
when n = 0
n = 1
n = 2
Ex.5 Identifythe reaction order fromeach of the following rate constant.
(i) k = 2.3 10
5
Lmol
1
s
1
(ii) k = 3 10
4
s
1
Order for k = 2.3 10
5
Lmol
1
s
1
is 2 and order for k = 3 10
4
s
1
is 1 fromunits. Sol.
6. DISTINCTION BETWEEN RATE OF REACTION AND RATE CONSTANT:
Rate of a reaction :
(i) The rate of reaction at anyinstant of time depends uponthe molar concentrations of the reactants at
that time.
(ii) Its units are always mole litre
1
time
1
.
Rate constant :
(i) The rate constant is constant for a particular reaction at a particular temperature and does not
depend upon the concentration of the reactants.
(ii) Its unit depends upon the order of reaction.
5
Chemical Kinetics
7. RATE LAW :
(i) It mayalso not depend upon the concentration of each reactant or product of the reaction.
Suppose,mA+ nBProduct
(ii) Rate of a chemical reaction is directlyproportional to the concentration of reactants.
(iii) The rate lawrepresents the experimentallyobservedrate of reactionwhichdepends upon the slowest
step of the reaction.
(iv) Rate lawcannot be deduce fromthe equation for a given reaction. It can be find by experiments
only.
(v) The rate lawmaynot bear a simple relationship for the stoichiometric equation.
(vi) It may not depend upon the concentration of species, which do not appear in the equation for the
over all reaction.
8. ORDER OF REACTION :
The order of a reaction may be defined as the sumof the powers to whichconc. terms must be raisedin
anexperimentally determined differential rate equation :
For the reaction : mA + nBproduct
Experimental rate equation : r = k [A]
p
[B]
q
order with respect toA= p
order withe respect to B= q
Total order = p + q
Note: (i) Order maybe zero, fractional, integer or negative.
(ii) p and q may be equal to mAand mB.
Ex.6 Reaction
H
2
+ Cl
2
2HCl
H
2
+ Br
2
2HBr
H
2
+ I
2
2HI
Exp. rate equ.
r = k [H
2
] [Cl
2
]
order
0
3/2
2
0 0
1/2
r = k [H
2
] [Br
2
]
r = k [H
2
] [I
2
]
Ex.7 Reaction : CO(g) + Cl
2
(g) COCl
2
(g)
r = k [CO]
2
[Cl ]
1/2
order = 2.5
2
COCl
2
(g) CO(g) + Cl
2
(g)
Reaction :
3/2
r = k [COCl
2
] order = 1.5
For a reaction, A+ B Product; the rate law is given by, r = k[A]
1/2
[B]
2
. What is the order of the
reaction ?
Ex.8
1 1
Sol.
Ex.9
Order of the reaction = + 2 = 2
2
or 2.5.
2
The conversion of the molecules XtoYfollows second order kinetics. If the concentration Xis increased
to three times, howwill its affect the rate of formationofY?
For the reaction, X Y, as it follows second order kinetics, the rate lawequation will be
Rate = k [X]
2
If concentration of Xis increased to three times, then
Rate = k [3X]
2
or Rate = 9k [X]
2
Thus, the rate of reaction will become 9 times. Hence, the rate of formationofYwill increase 9 times.
Sol.
6
R [A]
m
m[B]
n
Chemical Kinetics
9. MOLECULARITYOFAREACTION :
Molecularityis defined as the number of molecules, atoms, or radicals that must collide simultaneously
in order for the reaction to take place.It is always a whole number and cannot be negative.
In the elementary processes :
Participating species Molecularity
One species participates..... unimolecular, 1
Two species participates..... bimolecular, 2
Three species participates....trimolecular, 3
Molecularity cannot be zero, fractional or more than three. Note:
Ex.10 N
2
O
4
2NO
2
......
H
2
+ I
2
2HI ......
unimolecular
bimolecular
(molecularity =1)
(molecularity =2)
(molecularity =3) 2FeCl
3
+ SnCl
2
2FeCl
2
+ SnCl
4
...... trimolecular
Note : If the reaction takes place in two or more steps thenthe overall molecularity of the reaction is
monitored bythe slowor rate determining step.
TYPEOFREACTIONS
Onthe basis of number of steps involved in the reaction, they are classified as elementary or complex
reactions.
10. ELEMENTARY REACTIONS :
These are single step reactions. For such reactions, order andmolecularity are always same. In another
words, rate lawand the lawof mass action will have same expression.
11. COMPLEX REACTIONS :
These are multi-step reactions. For such reactions, order and molecularity may or may not be same.
- Insuchreactions, some intermediates are formed. Intermediates are the speciesdifferent thanreactants
as well as products.
Each step of suchreaction is elementary reaction.
The overall rate of reaction will be equal to the rate of slowest step. This is why, the slowest step is
called the rate determining step (RDS) of reaction. Areaction can have more than one RDS,
The overall molecularity of reaction is the molecularity of the RDS. However, it has no significance.
The order ofreactionmayor maynot be the overall order of reaction. If depends on the concentration
terms involved inthe rate lawexpression of RDS. If it is not overall order, it may be determined by
equilibriumapproach or be steady state approximation.
-
-
-
-
12. EQUILIBRIUMAPPROACH :
The concentration ofintermediate can be determinedfromequilibriumconstant of the reaction involved.
For example, let the mechanism of reactionA+ 2 BC+ Dis
k
1
Step I: A+ B I
k
1

k
2

Step II: I + B C + D
Now, the overall rate of reaction, r = rate of step II = k
2
[I][B]
...(i)
7
Chemical Kinetics
It cannot be the correct rate lawfor the overall reaction because the overall rate of reaction shouldbe in
terms ofconcentrations ofAandB(reactants). The concentrationofI canberelatedwiththe
concentrations ofAandBwith the help of first equilibrium.
k
1
[I ]
For step I, equilibrium constant, K
eq
= =
k [A][B]
1
k
1
[I] = [A][B]
k
1
k
1 2
Putting this value in equation (i), r = k
2

k
[A][B][B] = k[A][B]
,
1
k
1
where, k =
k
1
Hence, the overall rate of reaction is 1 + 2 = 3
13. STEADY STATE APPROXIMATION :
In this method, we assume that the intermediates formed are so reactive that after some time from
initiation of reaction (called induction period), the net rate of their formation becomes zero. Theyreact
with the same rate of their formation. For example, let the mechanismof reaction A+2 BC+D
is
k
1
Step I: A+ B I
k
1

k
2

Step II: I + B C + D
d[C]
The rate of reaction maybe given as r = + = k
2
[I][B] ...(ii)
dt
d[I ]
Now, fromsteady state approximation on the intermediate, I + = 0
dt
or, k
1
[A][B] k
1
[I] k
2
[I][B] = 0
k [A][B]
1
or, [I] =
k
1
+ k
2
[B]
k k [A][B]
2
k
1
[A][B]
1 2
Putting this value in equation (ii), r = k
2

k
[B] =
[B] + k k + k [B]
1 2 1 2
- Steadystate approximation gives better result whenthe intermediate is less stable while equilibrium
approach gives better result when the intermediate is more stable. More stable intermediate reacts
very less and hence, the concentration of intermediate at any time remains nearly equal to its equilibrium
concentration.
- When conditions of equilibrium approach are applied on the result obtained fromsteady state
approximation, the same rate law expression will come. For the above reaction, step I is faster than
step II. As step I is at equilibrium,
r
1
= r
1
>> r
2
or, k
1
[I] >> k
2
[I][B]
k
1
>> k
2
[B]
k
1
+ k
2
[B] ~ k
1
or,

k k [A][B]
2
~ [A][B]
2
,
2 1
Now, the rate expression obtained fromsteady state is r =
k
1
+ k
2
[B]
which is exactly same expressionobtained fromequilibriumapproach.
8
Chemical Kinetics
14. TRUE AND APPARENT RATE CONSTANT :
For acidic hydrolysis of ester:
H
+
RCCOR' + H
2
O RCOOH + R'OH

+
the rate of reaction, r = k[RCOOR'][H
2
O][H ], where k is the true rate constant of the reaction,
Now, H
+
is the catalyst of reaction and hence, throughout its concentration will remain constant. H2Ois
solvent for the reaction and taken in large excess and hence, its concentration will also remain constant.
Hence, k[H
2
O][H ] = constant, called apparent rate constant of the reaction.
15. PSEUDO UNIMOLECULAR REACTION :
+
H
Consider the reaction : CH
3
COOC
2
H
5
+ H
2
O CH
3
COOH+ C
2
H
5
OH
Since water is present in large excess, its concentration hardlychanges during the course of the reaction.
And as such rate depends onlyonthe concentration of ester. The order is one but the molecular is two.
Such reactions are called pseudo unimolecular reaction.
The reaction 2NO+ Br
2
2NOBr, obeys the following mechanism:
Ex.11
Fast
Step I: NO + Br
2
NOBr
2
Slow
Step II: NOBr
2
+ NO 2NOBr
Suggest the rate expression.
Step II is the rate determining step of the reaction and hence, r =
k[NOBr
2
] [NO]
However, NOBr
2
is an intermediate and thus its concentration should be determined fromthe equilibrium
of step I.
[NOBr
2
]
Sol.
For step I, equilibrium constant, k
eq
=
[NO][Br ]
2

[NOBr
2
] = k
eq
[NO][Br
2
] ... (ii)
2
Thus, by equations (i) and (ii), r = k .k
eq
[NO] [Br
2
]
r = k' [NO]
2
[Br ]. where k' = k.k
or,
2 eq
Ex.12 The following mechanisms are proposed for the reaction
CO + NO
2
CO
2
+ NO at low temperature:
(a) 2NO
2
N
2
O
4
(fast)
(slow)
(slow)
(fast)
N
2
O
4
+ 2CO 2CO
2
+ 3NO
(b) 2NO
2
NO
3
+ NO
NO
3
+ CO NO
2
+ CO
2
d[CO
2
]
2
which of the above mechanism are consistent with the observed rate law: +
For mechanism(a), r = rate of step II
= k[NO
2
] ?
dt
Sol.
2
= k[N
2
O
4
][CO] ...(i)
[ N
2
O
4
]
Now, fromstep I : k
eq
=
]
2
[NO
2
2
or, [N
2
O
4
] = k
eq
[NO
2
]
2 2
Hence from(i), r = k.k
eq
[NO
2
] [CO] , but it is not the given rate law
Hence, the mechanismis not consistent with the rate law.
2
For mechanism (b),
r = rate of step, I = k[NO
2
] , which is the given rate law.
Ex.13
For a hypothetical reaction
A+ Bproducts, the rate lawis, r = k [B] [A], the order and molecularity of reaction is :
Order = 1 + 0 = 1, molecularity = 1 + 1 = 2 Sol.
9
Chemical Kinetics
16. TYPES OF REACTIONS BASED ON ORDER OF REACTION :
(I) Zero Order Reactions :
Zero order reactionmeans that the rate of the reaction isproportional to zero power of the
concentration of reactants. Consider the reaction,
R P
d[R]
Rate = = k [R]
0
dt
As anyquantityraised to power zero is unity
d[R]
Rate = = k 1
dt
d[R] = k dt
Integrating bothsides
[R] = k t + I
where, I is the constant of integration.
...(1)
At t =0, the concentration of the reactant R= [R] , where [R] is initial concentration of the reactant.
0 0
Substituting inequation (1)
[R] = k 0 + I
0
[R] = I
0
Substituting the value of I inthe equation (1)
[R] = kt + [R] ...(2)
0
O
Time (t)
Comparing withequation of a straight line, y=mx+c, if we plot [R] against t, we get a straight line with
slope = k and intercept equal to [R] .
0
Further simplifying equation (2), we get the rate constant, k as
[R]
0
[R]
k = ...(3)
t
Zero order reactions are relatively uncommon but theyoccur under special conditions. Some enzyme
catalysed reactions and reactions with occure on metal surfaces are a few examples of zero order
reactions. The decomposition of gaseous ammonia ona hot platinumsurface is a zero order reaction at
highpressure.
1130K
2NH (g) N (g) + 3H (g)
3 Pt catalyst 2 2
Rate = k [NH ]
0
= k
3
In this reaction, platinummetal acts as a catalyst. At highpressure, the metal surface gets saturated with
gas molecules. So, a further change in reaction conditions is unable to alter the amount of ammoniaon
the surface of the catalyst making rate of the reaction independent of its concentration. The thermal
decomposition of HI on gold surface is another example of zero order reaction.
10
C
o
n
c
e
n
t
r
a
t
i
o
n
o
f
R
K = - slope
Chemical Kinetics
Note: (i) Reaction betweenAcetone and Bromine
(ii)Dissociation of HI ongold surface
(A) Unit of Rate Constant
[R]
0
[R]
k =
t
Unit of rate of reaction =Unit of rate constant.
(B) Half-life period (t
1/2
) - The time in which half reaction is
completed
R
0
At t = t
1/2
; R =
2
t
1/ 2
R
0
or 1/ 2
The half life period is directly proportional to the initial concentration of the reactants.
Ex.14 The reaction 2A+ B + C D + 2E; is found to be first order inA; second order in B and zero order in C.
(i)
(ii)
(i)
Give the rate law for the above reaction in the form of a differential equation.
What is the effect on the rate of increasing the concentration of A, B and C two times?
The rate lawaccording to given information maybe given as, Sol.
dx
= K[A]
1
[B]
2
[C]
0
dt
(ii) When concentration ofA, Band Care doubled thenrate will be
dx
= K[2A][2B]
2
[C]
0
dt
=8K[A][B]
2
[C]
0
i.e., rate becomes 8 fold, the original rate.
(II) First Order Reactions :
In this class of reactions, the rate of the reactionis proportional to the first power of the concentration of
the reactant R. For example,
R P
d[R]
d[R]
Rate = = k [R] or = kdt
dt
[R]
Integrating thisequation, we get ln [R] = kt + I ...(4)
Again, I is the constant of integration and its value can be determined easily.
When t =0, R= [R] , where [R] is the initial concentration of the reactant.
0 0
Therefore, equation (4) can be written as
ln [R] = k 0 + I
0
ln [R] = I
0
Substituting the value of I inequation (4)
ln [R] = kt + ln[R] ...(5)
0
Rearranging this equation
[R]
ln = kt
[R]
0
[R]
0 1
or k = ln ...(6)
t [R]
11
t =
R
0
2K
K mol lit
-1
sec
-1
Chemical Kinetics
At time t fromequation (4)
1
ln[R] = kt + ln[R]
...(7)
1 1 0
At time t
2
ln[R] = kt + ln[R] ...(8)
2 2 0
where, [R] and [R] are the concentrations of the reactants at time t and t respectively.
1 2 1 2
Subtracting (8) from(7)
ln[R] ln[R] = kt (kt )
1
[R]
1
ln
2 1 2
= k (t t )
[R]
2
2 1
[R]
1
1
k = ln ...(9)
(t
2
t
1
) [R]
2
Equation (5) can also be written as
[R]
1
ln = kt
[R]
2
Taking antilog of bothsides
[R] = [R] e
kt
0
Comparing equation (5) with y=mx + c, if we plot ln [R] against t (Fig. 4.4) we get a straight line with
slope =k and intercept equal to ln [R]
0
The first order rate equation (6) can also be written in the form
[R]
0 2.303
k = log ...(10)
[R]
t
[R]
0
kt
log =
[R] 2.303
If we plot a graph between log [R] / [R] vs t,
0
the slope = k/2.303
Hydrogenation of ethene is an example of first order reaction.
C H (g) + H (g) C H (g)
2 4
Rate = k [C H ]
2 2 6
2 4
All natural and artificial radioactive decay of unstable nuclei take place byfirst order kinetics.
O t
Fig. :Aplot between ln[R] and t
for a first order reaction
Fig. : Plot of log[R] /[R] vs time for a first
0
order reaction
226
Ra
4
He +
222
Rn Note: (i)
88 2 86
Rate = k [Ra]
(ii)
(A)
Decomposition of N O and N Oare some more examples of first order reactions.
2 5
Unit of Rate Constant :
2
2.303 [R
0
]
log K =
t [R]
K = sec
1
12
l
n
[
R
]
ln[R
0
]
k = slope
Chemical Kinetics
R
0
(B) Half-life period (t
1/2
) : At t = t
1/2
, R = ,
2
=
2.303
R
0
2.303 0.693
log 2
K = log =
t
1/ 2
R
0
/ 2 t
1/ 2
t
1/ 2
0.693
t =
K
1/2
Some Examples of First Order reaction andTheir Rate constant
1. For Gas Phase Reaction
Let us consider a typical first order gas phase reaction
A(g) B(g) + C(g)
Let p be the initial pressure ofAand p the total pressure at time 't'. Integrated rate equation for such a
i
reaction can be derived as
t
Total pressure p = p + p + p (pressure units)
t A B C
p , p and p are the partial pressures ofA, Band C, respectively.
A B C
If x atmbe the decrease in pressure ofAat time t and one mole each of Band Cis being formed, the
increase in pressure of Band Cwill also be x atmeach.
A(g)
p atm
B(g)
0 atm
xatm
+ C(g)
0 atm
xatm
At t = 0
At time t
i
(px) atm
i
where, p is the initial pressure at time t = 0.
i
p = (p x) + x + x = p + x
t i
x = (p p )
i
t i
where, p = p x = p (p p ) = 2p p
A i
| 2.303 |
|
i t i i t
| p
|
log
i
| k =
\
...(11)
t p
A . .
\
p
i 2.303
= log
(2p p )
i t
t
2. Pseudo Unimolecular Reaction : Inversion of cane sugar
H
+
C H O + H O C H O + C H O
12 22 11 2
Sucrose
6 12 6 6 12 6
Glucose Fructose
The progress of the reactioncanbe studiedusingapolarimeter. Cane sugar andglucose are dextrorotatory
while fructoseis laevorotatary
If u is polarimetric reading at zero time.
0
u is polarimetric reading after time t.
t
and u is polarimetric reading after infinite time

2.303
log
u
0
u

k =
t u
t
u

3. (a) Decomposition of H O
2 2
1
H O
Pt
H O + O
2 2 2
2
2
The progress of the reaction can be studied either bymeasuring the volume of oxygen gas after different
intervals of times or bytitrating a definite amount of reaction mixture with standard KMnO at different
4
intervals of time.
If V and V represent the volumes of KMnO used at the start of the reactions and at any time t,
0
respectively
t 4
R V , R V
0 0 t
2.303 V
0
Therefore, k = log
V t
t
13
Chemical Kinetics
(b) Decomposition of ammoniumnitrite : NH NO N + 2H O
4 2 2 2
If V is thevolume of N evolved at anytime t andV is the volumeof N evolvedwhenthe decomposition
t
is complete, then,
2 2
V
2.303
log k =
V

V
t t
4. Hydrolysis of ethyl acetate (ester)
H
+
CH COOC H + H O CH COOH + C H OH
3 2 5 2 3 2 5
Kinetics of this reaction is studied by titrating a definite volume of the reaction mixture with standard
alkali solution.
If V , V and V are volumes of standard alkali needed to neutralise a definite amount of x will be
0 t
proportional to V V and a will be proportional to V V .
t 0 0
2.303
log
V

V
0
Hence, k = .
t V V
t
5. Oxide layer formation
1 t
max
k = ln
t t
max
t
where
t =Thickness of oxide layer after times
max
t =Thickness of oxide layer at time 't'.
6. Bacterial Growth:
1
ln
a + x
k =
t a
Second order reactions: When the rate of a reaction is determined by variation of two concentration terms, the
reaction is said to be of second order. For a general reaction,
A + B products, the rate law may be,
dx
= k [A]
2
[B]
0
, or ... (x)
dt
dx
= k [A]
0
[B]
2
, or ... (xi)
dt
dx
= k [A] [B] ... (xii)
dt
Thus, the rate of a second order reaction varies directly as the square of the concentration of reactant.
Unit of the second order rate constant :
From equation (x) we get
dx
Mol / litre
1 1
k = = = lit. mol time .
dt [ A]
2
Time interval Mol.
2
/lit.
2
The unit of second order rate constant involves concentration as well as time.
Examples of Second Order Reactions :
373 K
1.
2.
3.
4.
5.
Conversion of ozone into oxygen at 373 K : 2O
3
3O
2
Thermal decomposition of nitrous oxide : 2N
2
O 2N
2
+ O
2
Thermal decomposition of chlorine monoxide at 200 C (473 K) : 2Cl
2
O 2Cl
2
+ O
2
200C
Alkaline hydrolysis of an ester (saponification) : CH
3
COOC
2
H
5
+ NaOH CH
3
COONa + C
2
H
5
OH
KCN alc.
Benzoin condensation,
C
6
H
5
CHO + OHC.C
6
H
5
C
6
H
5
CH(OH)CO.C
6
H
5
reflux
14
Chemical Kinetics
6. Isomerisation of ammonium cyanate into urea.
O
heat
NH
2
C NH
2
NH
4
CNO
Second order rate equation: All the second order reactions obey the following kinetic equation,
1 x

t a(a x)
k = .... (xiii)
where k = second order rate constant
a = initial concentration of reactant (s)
x = concentration of reactant converted into products after time t
t = time elapsed
Characteristics of a Second Order Reaction :
1
(i) When a graph is plotted between t and a straight line is obtained. On rearranging, equation
(a x)
1 1
1

(xiii).t = The slope of the line . From this k an be evaluated.


k(a x) ka
k
l/(a-x)
(ii) The value of second order rate constant depends upon the unit in which concentration of the reactant(s)
x
is expressed, because the value of will be changed when the unit of concentration is changed.
a(a x)
(iii) The half-life period (t
1/2
) of a second order reaction is inversely proportional to the initial concentration
of reactant.
Fromexpression(xiii)
1 x

t a(a x)
k = ... (xiii)
when, t = t
1/2
, x = a/2
1 a / 2

so, k =
t
1/ 2
1
a(a a / 2)
or, t
1/2
= ... (xv)
ka
Thus, it is evident from equation (xv) that half-change time for a second order reaction is inversely proportional to
the initial concentration of reactant.
17. MISCELLENEOUS REACTIONS :
(a) Parallel Reactions
B
ln
[A]
0
[B]
=
k
1
A
= (k + k )t
[A]
t
[C] k
2
k
2
[A]
0
1 2
C
k
1
[A]
0
[A] = [A] e
kt
[B] = (1 e
kt
) [C] = (1 e
kt
)
k
1
+ k
2
k
1
+ k
2
0
15
T
i
m
e
Chemical Kinetics
(b) Consequtive Reaction
A

k
1

k
2

B C
|e e |
k
1
[A]
0 k
1
t k
2
t
[A] = [A]
e
k
1
t
; [B] =
(k k )
t 0 t
2 1
k
2
| k
|
k
2
k
1
|
k
1
[C] = [A] ([A] + [B] ) ; [B] = [A] .
t 0 t t max 0
\ 2 .
conc.
[C]
1 K
n
1
[B]
[A]
t =
max
K
1
K
2
K
2
time
(c) Reversible reaction
Consider the reaction
k
r
B A
k
b
At time t = 0
At time t = t
a
a x
a x
b
x
x At time t = t
aq eq eq
| | x x
k
eq

eq
|
f
=
ln

x
= (k + k )t ;
x
|
a x k f b
\ .
eq eq b
Table of Formulae
16
[
Chemical Kinetics
Important Graphical Representation
Zero Order First Order SecondOrder ThirdOrder
Ex.15 Time required to decompose SOCl to half of its initial amount is 60minutes. If the decompositionis a
2 2
first order reaction, calculate the rate constant of the reaction.
For a first order reaction, Sol.
0.693 0.693
k = = = 1.155 10
2
s
1
.
t
1/ 2
60 minutes
Ex.16 Afirst order reaction has a rate constant 1.15 10
3
s
1
. Howlong will 5 gof this reactant take to reduce
to 3 g ?
Given : [A] = 5 g, [A] = 3 g, k = 1.15 10
3
s
1
. Sol.
0
Applying first order kinetics equation and substituting the values, we get
[A]
0
2.303
t = log
[A]
t
2.303
log
5g
= = 2.00 10 (log 1.667) s
3
1.1510
3
3g
= 2.0 10
3
0.2219 s = 443.8 s
17
Chemical Kinetics
Ex.17 Afirst order reaction has a specific reaction rate of 10
2
s
1
. How much time will it take 10 g of its
reactant to be reduced to 2.5 g ?
Rate constant, k = 10
2
s
1
Initial reactant conc., [A] =10 g
Sol.
0
Final reactant conc., [A] =2.5 g
t
Time required, t = ?
For a first order reaction,
2.303
log
[A]
0
2.303
log
10 g
t = = = 2.303 10
2
log 4 s = 230.3 0.6020 s = 138.6 s.
10
2
s
1
k [A]
t
2.5g
Ex.18 Areaction that is of first order with respect to reactant Ahas rate constant 6 min
1
. If we start with
[A] = 5.0 mol L
1
, when would [A] reach the value of 0.5 mol L
1
?
Rate constant = 6 min
1
[A] = 5.0 mol L
1
Sol.
0
[A] = 0.5 mol L
1
t
t = ?
For a first order reaction,
1
2.303
log
[A]
0
2.303
5mol L
log
t = =
[A]
t
6 min
1
0.5 mol L
1
k
2.303
= log 10 min = 0.39 min.
6
Ex.19 For a first order reactions, it takes 5 minutes for the initial concentration of 0.6 mol L
1
to become 0.4
mol L
1
. Howlong in all will it take for the inital concentration to become 0.3 mol L
1
?
For a first order reaction,
2.303
log
[A]
0
Sol.
k =
k [A]
t
We have, [A] = 0.6 mol L
1
[A] = 0.4 mol L
1
t =5 min
0 t
So,
2.303 0.6 2.303 2.303 0.1761
k =
For,
log = log 1.5 min =
1
min = 8.1 10 min
1 2 1
5min 0.4 5 5
[A] = 0.3 mol L
1
t
2.303 2.303 2.303
log
[A]
0
0.6
log t = = = log 2 min = 8.5 min.
[A]
t
8.110
2
0.3 8.110
2
k
Ex.20 The following data were obtained duringthe first order thermal decomposition of NO (g) at constant
2 5
volume:
2N O (g)
2N O (g) + O (g)
2 5 2 4
Time /s
0
100
2
S.No.
1.
2.
Total Pressure/(atm)
0.5
0.512
Calculate the rate constant.
18
Chemical Kinetics
Sol. Let the pressure of N O (g) decrease by 2x atm. As two moles of N O decompose to give two moles
2 5 2 5
of N O (g) and one mole of O (g) the pressure of N O (g) increases by 2x atmand that of O (g)
2 4
increases byx atm.
2 2 4 2
2N O (g) 2N O (g) + O (g)
2 5
0.5 atm
(0.5 2x) atm
2 4
0 atm
2x atm
2
0 atm
xatm
Start t = 0
At time t
p
N O
2 5
p = + p + p
N
2
O
4
O
2
t
= (0.5 2x) + 2x + x = 0.5 + x
x = p 0.5
t
= 0.5 2 (p 0.5) = 1.5 2p
t
At t = 100 s; p = 0.512 atm
t
t
p
N O
2 5
= 1.5 2 0.512 = 0.476 atm
Usingequation(11)
2.303 p
i
2.303 0.5atm
k =
log
p
=
100s
log
0.476atm t
A
2.303
= 0.0216 = 4.98 10 s .
4 1
100s
Ex.21 For a reaction 2A Product, it is found that the rate of reaction becomes 2.25 when the
concentration ofAis increased 1.5times, calculate the order of reaction.
r = k [A]
n
Sol. Rate, .... (1)
When concentration is increased 1.5 times, the rate of reaction become 2.25.
2.25 r = k [1.5A]
n
.... (2)
k[1.5A]
n
2.25r
Divide (2) by (1) =
k[ A]
n
r
2.25 = (1.5)
n
(1.5)
2
= (1.5)
n
n = 2
18. DETERMINATION OF ORDER OF REACTION :
(I) Integration Method
Inthis method, value of Kis determined byputtingvalues of initial concentration ofreactants andchange
in concentration with time inkinetic equation of first, secondand third order reactions. The equation by
which constant value of Kis obtained is called order of that reaction.
R
0 2.303
K
1
= log (For first order reaction)
t
R
x 1
K
2
= (For second order reaction)
a(a - x) t
x(2a - x) 1
K
3
= (For third order reaction)
a
2
(a - x)
2
2t
19
Chemical Kinetics
(II) vant Hoff Differential Method
vant Hoff gave the following relationship between velocityVof n
th
order reactionand concentration of
reactants, C.
log(V
1
/ V
2
)
n =
log(C
1
/ C
2
)
C
1
and C
2
are two different concentrations, while V
1
and V
2
are their velocities.
Graphical Method
If a straight line is obtained ondrawinga graph between log (a x) andtime or
dt
, thenit is first order
(III)
dx
reaction.
If a straight line is obtained on drawing a graph between (a x)
2
and
dx
, then it is second order
dt
reaction.
Half-life Method (IV)
1
t
1/2

Relation between half-life period of a reaction and initial concentration is as follows :


a
n 1
For first order reaction (Half life a)
For second order reaction (Half life 1/a)
For thirdorder reaction (Half life 1/a
2
)
OstwaldIsolationMethod
This method is used to findout the order of complex reactions. If nA, nBand nCmolecules of substance
A, Band C, respectively, are present in a reaction, thennA+ nB+nCwill be the order of reaction.
When Band Care in excess, the order of reaction will be nA.
WhenAand Bare in excess, the order of reaction will be nC.
WhenAand Care in excess, the order of reaction will be nB.
TEMPERATURE EFFECT :
The rate of reaction is dependent on temperature. This is expressedin terms of temperature coefficient
which is a ratio of two rate constants differing by a temperature of 10
0
. Generally the temperature
selected are 298Kand 308K. It is mathematically expressedas,
Temperatur e coefficient =
rate constant at 308K
(V)
19.
rate constant at 298K
According to collisiontheory the reaction rate depends oncollision frequencyand effective collisions.
For a molecule to have effective collision it should fulfill two conditions :
(i) Proper orientation (ii) Sufficient energy
20. TEMPERATURE DEPENDENCE OF RATE CONSTANTS :
Arrhenius Equation the temperature dependence of the rate of a chemical reaction can be
accuratly explainbyArrhenius
Ae
E
a
/ RT
eq. k = ...(i)
where k =Arrhenius factor, E =Activation energy, R= Gas Constant, T=Temperature
a
20
Chemical Kinetics
For most reactions, the rate constant increases as the temperature increases. These rate constants in
most of the cases varywith temperature according to theArrehenius equation :
The parametersAand E for a given reactionare collectively calledArrhenius parameters. E iscalled the
a a
1
activation energy. Value of E is determined fromthe graph for and ln k determined experimentally.
T
a
Value ofAinturn is calculated once E is known.
a
In theArrhenius equation (i) the factor
e
Ea / RT
corresponds to the fraction of molecules that have
kinetic energygreater than E .
a
Taking natural logarithmof bothsides of equation(i) we get
E
a
ln k = lnA ...(ii)
RT
The plot of ln k vs 1/Tgives a straight line according to the equation (ii) as shownin(Figure below).
slope = E
a
/R
ln k
0 1/T
Aplot between ln k vs 1/T
Thus, it has been found fromArrhenius equation (i) that increasing the temperature or decreasing the
activation energy will result in an increase in the rate of the reaction and an exponential increase in the
rate constant.
E
a
In the plot, slope = and intercept = lnA. So we can calculate E andAusing these
values.
a
R
At temperature T , equation(ii) is
1
E
a
ln k = + lnA ...(iii)
RT
1
1
At temperature T , equation(ii) is
2
E
a
ln k = + lnA ...(iv)
2
RT
2
(sinceAis constant for a given reaction)
k and k are the values of rate constants at temperatures T andT respectively.
1 2 1 2
Subtracting equation (iii) from(iv), we obtain
E
a
E
a
ln k ln k =
2 1
RT RT
1 2
E
a
1 ( 1 k
2
ln =

(
R

T
1
T
2
k
1
E
a
1 ( k
2
log =
1

(
k
1
2.303R

T
1
T
2
21
Intercept = ln A
Chemical Kinetics
Ex.22
Sol.
What will be the effect of temperature onrate constant ?
Rate constant of a reaction is nearlydoubled withrise intemperature by10. The dependece of therate
constant on temperature is given byArrhenius equation, k =Ae
E
a
/ RT
, whereAis called frequency
factor and E is the activation energy of the reaction.
a
The rate of the chemical reaction doubles for an increase of 10 Kin absolute temperature from298 K.
Calculate E .
Ex.23
a
Here we are given that
When T = 298 K, k = k (say)
Sol.
1 1
When T = 308 K, k = 2k
2 2
E | T T |
log
k
2
a

2 1
|
=
2.303R

TT
|
k
1
\ 1 2 .
Substituting these values inthe equation, we get
E
a
308K 298K
log
2k
=
2.3038.314 J K
1
mol

1
298K308K
k
E
a
10
or log 2 =
2.3038.314 298308
E
a
10
or 0.3010 =
2.303 8.314 298308
0.3010 2.3038.314 298308
or E =
a
10
or E = 52897.8 J mol
1
= 53.6 kJ mol
1
.
a
Ex.24 Howthe value of activation energyis calculated fromthe rate constants at twodifferent temperatures. If
the value of activation energy is 50kJ/mol showthat by increasing the temperature from300 Kto 310
Krate constant becomes nearly double.
We knowthat Arrhenius equation can be written as
E
a
1
Sol.
log k = logA
2.303 R T
If k and k are the rate constants at two different temperature T andT thenArrhenius equation at both
1 2 1 2
the temperatures can be written as
E
a
1
log k = logA
1
2.303 R
E
a
T
1
1
log k = logA
2
2.303 R T
2
Substracting eq. (i) from(ii), we get
1 ( E
a
1

log k log k =
(
2 1
2.303 R

T
1
T
2
k
2
E
a
1 ( 1
or log =
(
k
1
2.303 R

T
1
T
2
22
Chemical Kinetics
Byknowing the values of k and k at temperature T andT value of E can be calculated.
1 2 1 2 a
Given T = 300 K, T = 310 K, E = 50 kJ or 50,000 J
1 2 a
Put these values in eq. (iii)
k
2
50000 1 1 (
log =
k
1
2.303 8.314

300 310
(

50000 310 300 (
=

310 300
(
2.3038.314
log
k
2
= 0.2808 or
k
1
=Antilog 0.2808 = 1.91 ~ 2
k
2
k
1
It is clear that by increasing the temperature from300 Kto 310 Krate constant becomes double.
Afirst order reaction is 50%complete in 30 minutes at 27Cand in 10 minutes at 47C. Calculate the
rate constants at 27Cand 47C, and the energy of activation of the reaction in kJ/mol.
Time for the completion of 50%reaction means t .
Ex.25
Sol.
1/2
It means t of the reaction at 27Cis 30 minutes and at 47Cis 10 min.
1/2
0.693
We knowthat k =
t
1/ 2
0.693
or k = = 0.0231 min
1
(at 27C)
30
0.693
and
We knowthat
k
log
2
k = = 0.0693 min
1
(at 47C)
10
1 ( E
a
1

=
(
2.303 R

T
1
T
2
k
1
k = 0.0693 min
1
, k = 0.0231 min
1
,
2 1
T = 27 + 273 = 30 K, T = 47 + 273 = 300 K,
1 2
R = 8.314 JK
1
mol
1
, E = ?
a
log
0.0693 E
a
1 1 (

=
(
0.0231 2.303 8.314
300 320

E
a
320 300 (
or log 3 =
2.303 8.314

300 320
(

E
a
20 (
0.4771 =
2.303 8.314

300 320
(

0.4771 2.3038.314300320(
or E =
( a
20
= 43848.49 J mol
1

E = 43.85 kJ mol
1
a
23
Chemical Kinetics
21. MECHANISM BASED ON INTERMEDIATE FORMATION :
For understanding it let us use the following simple reaction
H (g) + I (g) 2HI (g)
2 2
H I H I H I
+ +
H I H I H I
Formation of HI through the intermediate
For understanding the product formation inthe above reaction we highlight the following points:
According toArrhenius, this reaction can take place only whena molecule of hydrogen and a molecule
of iodine collide to formanunstable intermediate (Figure). It exists for a very short time and then breaks
up to formtwo molecules of hydrogen iodide.
- The energy required to formthis intermediate, called activated complex (C), is known as activation
energy (E ).
a
Figure is obtained byplotting potential energy v/s reaction coordinate. The reaction coordinate
represents the profile of energychange whenreactants change into products.
Activated
-
2HI
Reaction coordinate
Diagram showing plot of potential
energy vs reaction coordinate.
Some energyis released when the complex decomposes to formproducts. So, the final heat of the
reaction depends upon the nature of reactants and products.
All the molecules is the reacting species do not have the same kinetic energy.
-
-
Kinetic energy
Distribution curve showing energies
among gaseous molecules
22. ACTIVATION ENERGYAND THE RATE OF REACTION :
The molecules of the reacting substances must be promoted to the top of the energybarrier before these
are able to react. For this to happen, the molecules must absorb energy equal to the activation energy.
Depending upon the magnitude of the activationenergy, the following three cases are possible:
(i) When the activation energyis small. If the activation energyis low, thenlarger number of the reactant
molecules will be able to cross over the top of the energy-barrier. As a result, the reaction will be
faster. Thus, if the activation energyis low, then the reaction is fast.
(ii) When the activation energy is high, then only a fewmolecules would be able tocross the top of the
energybarrier. As a result, the reaction will be slow. Thus, if the activation energy of a reaction is
high, then the reaction is slow.
24
P
o
t
e
n
t
i
a
l
e
n
e
r
g
y
Most probablekinetic energy
complex
C
Activation
energy
A
H
2
+ I
2
B
Chemical Kinetics
(iii) Whenthe activation energy is zero, theneach molecule will be able to cross the top of the energy
barrier. As a result, the reactionwill be instantaneous, and almost explosive. Thus, if the activation
energyof a reaction is zero, then the reaction is instantaneous, (veryveryfast).
23. ACTIVATION ENERGIES OFA REVERSIBLE REACTION :
In a reversible reaction, the reactants react to form products, and the products react back to give the
reactant molecules. Thus, in a reversible reaction, there are two reactions proceeding in the opposite
directions. Each reaction has its own characteristic activation energy. So, in a reversible reaction there are
two activation energies: one for the forward reaction, and the other for the backward reaction. These are
commonlycalled as the energyof activation for the forward reaction (E ) andthe energyof activation
af
for the backward reaction (E ). Depending upon the relative energies of the reactants and products, the
a,b
following two cases are possible.
(i) When the energy of the products is lower than that of reactant
Whenthe energyof products is lower than that of the reactant, [Fig. (a)], then
Activation energyfor the forward reaction <Energy of activation for the backward reaction
or, E < E
af a,b
Therefore, the reaction inthe forward direction is faster than that in the backward direction.
E
r
E
R
E
p
Reaction coordinate
(Exothermic reaction)
(a)
(ii) When the energy of products is higher thanthat of the reactants
Whenthe energyof products is higher than that of the reactants [Fig (b)], then
Activation energyfor the forward reaction >Activation energyfor the backward reaction
or, E > E
af a,b
E
r
E
p
E
R
Reaction coordinate
(Endothermic reaction)
(b)
Thus, the reaction in the forward direction is slower than the reaction in the backward
direction.
It must be remarked here that the activation energyof a reaction in anydirection determines the speed
(rate) of reaction inthat direction. It does not sayanything about the extent of reaction in that direction.
Extent of reactioninany direction is governed bythe equilibriumconstant of the reaction.
25
E
n
e
r
g
y
E
n
e
r
g
y
E
a,f
E
a,b
Product
Reactant
E
a,f
E
a,b
Reactant
Product
Chemical Kinetics
24. ACTIVATION ENERGIESAND ENERGY CHANGE DURING REACTION:
In a reaction, the energy change is given by,
Energychange in a reaction =Energyof products Energyof reactants
or, AE = E E (i)
P R
According to the concept of activation energy, the reactant molecules in the forward reaction, and the
products molecules in the backward reaction must pass over the top of the energy-barrier (E ). Then,
T
one canwrite Eq.(i), in the following form
AE = E E + E E = (E E ) (E E )
(ii)
(iii)
P R T T T R
AE = E E
T P
or,
a,f a,b
Under constant pressure conditions, AE = AH,
So,
(a) When
AH = E E
a,f a,b
E < E
a,f a,b
AE = ve
AH = ve and,
Thus, whenthe activation energy for the forward reaction is less than that for the backward reaction,
energyis released during the course of reaction.
(b) When E > E
a,f a,b
AE = + ve
AH = + ve and,
Thus, whenthe activation energy for the forward reaction is more thanthat for the backward reaction,
energy is absorbed during the course of reaction.
25. MAXWELL'S- BOLTZMANN DISTRIBUTION CURVE :
Since it is difficult to predict the behavior of anyone molecule with precision, Ludwig Boltzmann and
James Clark Maxwell used statistics to predict the behaviour of large number of molecules. According
to them, the distribution of kinetic energy maybe describedbyplotting the fractionof molecules (N /N )
E T
with a given kinetic energy (E) vs kinetic energy (Figure). Here, N is the number of molecules with
E
energyE and N is total number of molecules.
T
s
nal
h
Kinetic energy
Distribution curve showing temperature
dependence of rate of a reaction
26
t
(t+10)
Energy of
activation This area show
fractionof additio
This area
cules whic
shows fraction
t at (t + 10
of molecules
reactingat t
mole
reac
Chemical Kinetics
The peak of the curve corresponds to the most probable kinetic energy, i.e., kinetic energyof maximum
fraction of molecules. There are decreasing number of molecules withenergies higher or lower thanthis
value. When the temperature is raised, the maximumof the curve moves to the higher energy value
(Figure) and the curve broadens out, i.e., spreads to the right such that there is a greater proportion of
molecules withmuch higher energies. The area under the curve must be constant since total probability
must be one at all times. We can mark the position of E onMaxwell Boltzmann distribution curve.
a
Increasing thetemperature of the substance increases the fractionof molecules, whichcollide withenergies
greater than E . It is clear fromthe diagramthat inthe curve at (T+10), the area showing the fraction of
a
molecules having energyequal to or greater thanactivation energygets doubled leading to doublingthe
rate of a reaction.
26. EFFECT OF CATALYST :
Acatalyst is a substance whichalters the rate of a reaction without itself undergoing any permanent
chemical change. For example, MnO catalyses the followingreactionsoas to increase itsrate considerably.
2
2KCl + 3O 2KClO
MnO

2
3 2
Catalytic Mechanism:
The actionof the catalyst can be explained by intermediate complex theory. According to this theory, a
catalyst participates in a chemical reactionbyforming temporarybonds withthe reactants resultinginan
intermediate complex. This has a transitoryexistence and decomposes to yield products and the catalyst.
It is believed that the catalyst provides an alternate pathway or reaction mechanism by reducing the
activation energy between reactants and products and hence lowering the potential energy barrier as
showninthe figure.
Energy of
activation
without
catalyst
catalyst
path with
Products
Reactioncoordinate
Effect of catalyst on activationenergy
It is clear fromArrhenius equation, k =A
e
E
a
/ RT
,
that the lower the value of activation energy faster will be the rate of a reaction.
Note: for catalysts
-
-
-
-
Asmall amount of the catalyst can catalyse a large amount of reactants.
Acatalyst does not alter Gibbs energy, AGof a reaction.
It catalyses the spontaneous reactions but does not catalyse non-spontaneous reactions.
It is also found that a catalyst does not change the equilibriumconstant of a reaction rather, it helps
in attaining the equilibriumfaster, that is, it catalyses the forward as well as the backward reactionto
the same extent so that the equilibriumstate remains same but is reached earlier.
27. COLLISION THEORY OF CHEMICALREACTIONS :
Though Arrhenius equation is applicable under a wide range of circumstances, collision theory, which
was developed byMax Trautz andWilliamLewis in1916-18, provides a greater insight into the energetic
and mechanistic aspects of reactions. It is based onkinetic theory of gases.
27
P
o
t
e
n
t
i
a
l
e
n
e
r
g
y
Reactionpath
without catalyst
Energy of
activation
Reactants
Reaction
with
catalyst
Chemical Kinetics
CollisionTheory :
According to this theory, the reactant molecules are assumed to be hard spheres andreactionis postulated
to occur whenmolecules collide with eachother. Various noteworthy points of collision theory are:
- Collision Frequency - The number of collisions per second per unit volume of the reaction mixture
is known as collision frequency(Z).
Activation Energy - Another factor which affects the rate of chemical reactions is activation energy
(as we have already studied).
Bimolecular Elementary Reaction -
For a bimolecular elementary reaction
A+ BProducts
rate of reaction can be expressed as
-
-
e
E
a
/ RT
Rate = Z ...(vi)
AB
e
E
a
/ RT
where Z represent the collision frequency of reactants, Aand B and represents the
AB
fraction of molecules withenergies equal to or greater than E.
a
28. THRESHOLD ENERGY AND ORIENTATION OF COLLIDING
MOLECULES :
The equation (vi) predicts the value of rate constants fairly accurately for the reactions that involve
atomic species or simple molecules but for complex molecules significant deviations are observed.
The reason could be that all collisions do not lead to the formation of products. The collisions inwhich
molecules collide with sufficient kinetic energy (called threshold energy*) and proper orientation, so as
to facilitate breaking of bonds between reacting species and formation of newbonds to formproducts
are called as effective collisions.
Note: The formation of methanol frombromoethane depends upon the orientation of reactant molecules as
showninthe figure. The proper orientationof reactant molecules lead to bond formationwhereas
improper orientation makes themsimply bounce back and no products are formed.
CH
3
Br + OH CH
3
OH + Br
H
Improper
Br + OH No products H C
H
H
H C Br + OH
H
C
H
H
HO C
H
Br
H
OH H + Br
Orientation
H
Intermediate
Steric Factor
To account for effective collisions, another factor P, called the probabilityor steric factor is introduced.
It takes into account the fact that ina collisionmolecules must be properlyoriented i.e.,
e
E
a
/ RT
Rate = PZ
AB
Note: for collision theory
- In collision theoryactivation energyand proper orientation of the molecules together determine the
criteria for an effective collision andhence the rate of a chemical reaction.
Collision theoryalso has certain drawbacks as it considers atoms/ molecules to be hardspheres and
ignores their structural aspect.
Comparing the equation (vi) withArrhenius equation, we can saythat A(ofArrhenius Equation) is
related to collision frequency.
-
-
28
Orientation
proper
Chemical Kinetics
SOLVEDEXAMPLES
Ex.1 The half-life period of a first order reaction is 30 minutes. Calculate the specific reaction rate of the
reaction. What fractionof the reactant remains after 70 minutes ?
0.6932 0.6932
Sol. k = = = 0.231 min
1
.
1
t
1
30
2
Let the reaction be
a
A
0
Product
x
Initial concentration

(a x) Concentration after 70minutes


(a x)
fraction of the reactant remained unreacted = .
a
Now,
2.303
log
a
k =
1
t a x
2.303
log
a
or 0.0231 =
70 a x
a 0.0231 70
log = = 0.7021.
a x 2.303
Taking antilog, we get
a
= 5.036.
a x
a x 1
= 0.2.
~
a 5.036
Ex.2 The specific reactionrate of a first-order reaction is 0.02 s
1
. The intial concentration of the reactant is 2
moles/litre. Calculate (a) initial rate, and(b) rate after 60 seconds.
We knowthat for a first-order reaction :
Rate of a reaction = k molar concentration of the reactant.
(a) inital rate =k initial concentration
= 0.02 2 = 0.4 mole/litre/second.
(b) Now, to calculate rate after 60 seconds, let us first calculate concentration of the reactant after 60
seconds.
2
Sol.
2.303
log
k = = 0.02.
concentration after 60seconds
60
concentration of the reactant after 60 seconds = 0.60 M.
rate after 60 seconds = k concentration of the reactant after 60 seconds.
= 0.02 0.60
= 0.012 mole /litre/second.
29
Chemical Kinetics
Ex.3 The rate constant is numericallythe same for three reactions of first, second andthird order respectively,
the unit of concentration being inmoles per litre. Whichreaction should be the fastest and is thistrue for
all ranges of concentrations ?
Suppose R , R and R are the rates of three reactions of first, second and third order respectively and Sol.
1 2 3
k is the rate constant, whichis the same for the three reactions,
R = k[A]
1
1
R = k[A]
2
2
R = k[A]
3
3
[A] being the concentration of the reactant Ain moles per litre.
Nowif, [A] = 1,
[A] < 1,
[A] > 1,
R = R = R ;
1 2 3
R > R > R ;
1 2 3
R < R < R . and
1 2 3
Ex.4 For the reaction 2NO+ Cl 2NOCl, it is found that doubling the concentration of both reactants
2
increases the rate by a factor of 8, but doubling the Cl concentrationalone, onlydoubles the rate. What
2
is the order of the reaction with respect to NOand Cl ?
2
Sol. Rate = k [NO]
m
[Cl ]
n
2
Let the concentrations of NOand Cl be xand yrespectively.
2
According to the question,
R = k x
m
y
n
1
R = k (2x)
m
(2y)
n
and,
2
= k . x
m
y
n
. 2
m + n
R
2
= 2
m + n
= 8 = 2
3
. (given)
R
1

Again,
m+ n = 3
R = k (x)
m
(2y)
n
3
= k x
m
y
n
. 2
n
.
R
3
= 2
n
= 2. (given)
R
1
n = 1
m = 3 1 = 2.
Ex.5 In a reaction 2N O 4NO + O , the rate can be expressed as
2 5 2 2

d[ N
2
O
5
]
(i) = k [N O ]
1 2 5
dt
d[NO
2
]
(ii) = k [N O ]
2 2 5
dt
d[O
2
]
(iii) = k [N O ]
3 2 5
dt
Howare k , k and k related ?
1 2 3
30
Chemical Kinetics
Sol. The rate lawof the given reaction is
1

d[N
2
O
5
] 1

d[NO
2
] d[O
2
]
rate =
2
= = = k[N O ]
2 5
dt 4 dt dt
d[N
2
O
5
]

= 2k [N O ] = k [N O ]
2 5 1 2 5
dt
d[ NO
2
]
= 4k [N O ] = k [N O ]
dt
d[O
2
]
2 5 2 2 5
= k [N O ] = k [N O ]
dt
2 5 3 2 5
k
1
= 2k

k
2
= 4k

k
3
= k


k
1
k
2
or k = = = k
3
2 4
or 2k = k = 4k .
1 2 3
Ex.6 The reaction, N O 4NO + O , is forming NO at the rate of 0.0072 mol/L/s at some time.
2 5 2 2 2
(a) What is the rate of change of [O ] at this time ?
2
(b) What is the rate of change of [N O ] at this time ?
2 5
(c) What is the rate of reaction at this time ?
The rate of the reaction is expressed as Sol.
1 d[N
2
O
5
] 1 d[NO
2
] d[O
2
]
rate =
2
= +
4
= +
dt dt dt
d[ NO
2
]
and given that =0.0072 mole/L/s.
dt
1
(a) Rate of appearance of O = rate of appearance of NO
4
2 2
d[O
2
] 1 d[ NO
2
]
=
dt 4 dt
1
= 0.0072 = 0.0018 mole/L/s.
4
1
(b) Rate of disappearnace of N O = rate of appearance of NO
2
1 d[NO
2
]
2 5 2
d[N
2
O
5
]
=
dt 2 dt
d[N
2
O
5
] 1
= 0.0072 = 0.0036 mole/L/s.
dt 2
1 d[ NO
2
] 1
= 0.0072 = 0.0018 mole / L/s. (c) Rate of reaction =
4 dt 4
31
Chemical Kinetics
Ex.7 The rate of a first order reaction is 0.04 mole/L/s at 10 minutes and0.03 mole/L/s at 20 minutes after
initiation. Find the half-life of the reaction.
Let the concentrations of the reactant after 10 min and 20 min be C and C respectively. Sol.
1 2
rate after 10 min = k.C = .04 60
1
and rate after 20 min = k.C = .03 60.
2
c
1
4
=
c
2
3
Suppose the reactionstarting after 10 minutes
2.303 2.303 c
1
4
k = log =
c
log = 0.02878
3 10 10
2
0.6932 0.6932
t = = = 24.086 min.
k 0.02878
1/2
Ex.8 For a reaction at 800 C, 2NO+ 2H N + 2H O, the following data were obtained :
2 2 2
d[NO]/dt 10
4
mole litre
1
min
1
4.4
2.2
0.24
[NO] 10
4
mole / litre
(i) 1.5
(ii) 1.5
(iii) 1.5
[H ] 10
3
2
mole / litre
7.0
3.5
2.0
What is the order of this reaction with respect to NOand H ?
2
Fromthe data (i) and (ii), we see that when the concentration of H is halved, the rate is also halved at Sol.
2
constant concentration of NO. Hence the reaction is of first order with respect to H . Let us now
2
consider the data (ii) and (iii) to determine the order with respect to NOas [H ] is constant.
2
The rate lawof the above reaction is

1

d[NO]
rate = = k[NO] [H ]
2
m 1
2 dt
where mis the order with respect to NO

d[NO]
or = 2k [NO] [H ]
2
m
dt
Substituting data (ii) and (iii), we get
2.2 10
4
= 2k (1.5 10
4
)m. (2 10
3
)
0.24 10
4
= 2k (0.5 10
4
)m. (2 10
3
)
Dividing (1) by(2),
...(1)
...(2)
(1.510
4
)
m
2.2 220
= = 3
m
or = 3
m
(0.510
4
)
m
0.24 24
Taking log, log 220 log 24 = mlog 3
2.4324 1.3802 = m 0.4771
0.9622 = 0.4771 m
0.9622
or
or m = = 2.
0.4771
Hence the reaction is of second and first order with respect to NOand H respectively.
32
Chemical Kinetics
For a chemical reactionA+ BProduct, the order is 1 with respect to each ofAand B. Find xand y
fromthe given data.
Ex.8
Rate (moles/ L/s)
0.10
0.40
0.80
[A]
0.20 M
x
0.40 M
[B]
0.05 M
0.05 M
y
Sol. The rate lawmay be written as
rate = k [A] [B]
Substituting the first set of data in the rate law, we get,
0.10 = k 0.20 0.05
k = 10.
Nowsubstituting the second and third sets of data, we get,
0.40 = 10 x 0.05
x = 0.80 M.
And, 0.80 = 10 0.40 y
y = 0.20 M.
Ex.9 The energy of activation and specific rate constant for a first-order reaction at 50 C
2N O 2N O + O
2 5
(in CCl )
2 4 2
(in CCl )
4 4
are 100 kJ/mole and 3.46 10
5
s
1
respectively. Determine the temperature at which the half-life of the
reaction is 2 hours.
Sol. Let us calculate the rate constant (say k ) at a temperature (say T ) at which t is given to be
2 2 1/2
2 60 60 seconds.
The rate constant at a temperature 298 K(sayT ) is given as 3.46 10
5
s
1
(say k )
1 1
0.6932
k = = 9.62 10
5
s
1
.
26060
2
E | T
2
T
1
| k
2

2.303R TT
|
Thus, log
k
=
\ 1 2 .
1
| T 298 |
9.62 10
5
log
3.4610
5
100000
2
|
=
|
29T
2.303 8.314
\ 2 .
or T = 310 K.
2
Ex.10 InArrhenius's equation for a certain reaction, the value ofAand E (activation energy) are 4 10
13
s
1
and 98.6 kJ mol
1
respectively. If the reactionis of first order, at what temperature will its half-life period
be tenminutes ?
We have,
k =Ae
E/RT
E
Sol.
ln k = lnA
RT
E
2.303 log k = 2.303 logA
RT
33
Chemical Kinetics
E
or log k = logA ...(i)
2.303RT
Giventhat A= 4 10
13
s
1
, E = 98.6 kJ mol
1
t = 10 60 s.
1/2
0.6932 0.6932
For first-order reaction k = = = s
1
t
1/ 2 600
Thus (i) becomes,
98.6
0.6932
log = log (4 10 )
13
[R= 8.314 10 kJ/K/mol]
3
2.3038.31410
3
T 600
T = 311.2 K.
Ex.11 The activation energyfor the reaction,
O (g) + NO (g) NO (g) + O (g)
3 2 2
is 9.6 kJ/ mole. Prepare an activation energyplot it AH for this reaction is 200 kJ/mole. What is the
energyof activation for the reverse reaction ?
Energy of activation for reverse reaction
= 9.6 + 200 kJ
= 209.6 kJ.
Sol.
Reaction coordinate
Ex.12 Fromthefollowingdata for thedecompositionofdiazobenzene chloride, showthat thereactionisoffirst order
Time:
Vol. of N :
20
10
50
25
70
33

162
(min)
(mL)
2
Sol. C H N Cl C H Cl + N
6 5 2 6 5 2
Initial concentration
Concentration after time t
a
(a x) x
At time, i.e, when the reaction is complete, the whole of C H N Cl converts into N . Hence volume
6 5 2 2
of N at time corresponds to the inital concentration 'a' while volumes of N at different time intervals
2
correspond to xas shown above.
2
Inserting the given data inEquation of first order reaction, we get the following results.
2.303 162
For t = 20 min, k = log = 0.0032 min
1
20 162 10
1
34
E
n
e
r
g
y
9.6 kJ
E
reverse
200 kJ
Chemical Kinetics
2.303 162
For t = 50 min, k = log = 0.0033 min
1
1
50
2.303
162 25
162
For t = 70 min, k = log = 0.0032 min
1
1
70 162 33
The constancy of k shows tha the decomposition of C H N Cl is a first order reaction.
1 6 5 2
Ex.13 1 mLof methyl acetate was added to 20mLof 0.5 NHCl and 2mLof the mixture was withdrawn from
time t time duringthe progress of hydrolysis of the ester and titratedwith a solutionof alkali. The amount
c
of alkali needed for titration at various intervals is givenbelow:

42.03
Time:
Alkali used :
0
19.24
20
20.73
119
26.6
(min)
(mL)
Establish that the reaction is of first order.
CH COOCH + H O
HCl
[H
+
]
Sol. CH COOH + CH OH
3 3 2 3 3
Initial concentration
Concentration after time t
a
(a x) x
HCl acts as a catalyst. Since ineverytitration the amount of HCl is the same, the alkali used against HCl
is subtracted fromthe total alkali used (given in the data) to get the volume of alkali used onlyagainst
CH COOH.At zero time no CH COOHis formed, so alkali used at zero time is onlyfor HCl.
3
we thus have,
Time(min) :
Vol. of alkali
(mL) used
against CH COOH
3
0 20 119
: 19.24 20.73 26.6 42.03
3
19.24 = 0 19.24 =
1.49
(x)
19.24 =
7.36
(x)
19.24 =
22.79
(a)
Nowfollowing the same way as inExample 40, we get the following results :
2.303 22.79
For t =20 ; k = log = 0.0033 min
1
20
2.303
22.79 1.49
22.79
1
= 0.0032 min
1
For t =119 ; k = log
119 22.79 7.36
1
The constancy of k shows that the reaction is of first order.
1
Ex.14 The optical rotation of can sugar in0.5Nlactic acid at 25 Cat various time intervals are givenbelow:
Time(min) :
Rotation() :
0
34.50
1435
31.10
11360
13.98

10.77
Showthat the reaction is of first order.
Lactic acid
Sol. C
12
H
22
O
11
+ H
2
O C
6
H
12
O
6
+ C
6
H
12
O
6
Sucrose (excess) Glucose
Dextro
rotatory
Fructose
Laevo
rotatory
Dextrorotatory
35
Chemical Kinetics
Since inthis reactiondextro formchanges to laevo form, the optical rotation decreases with the progress
of the reaction. Thus change inrotationis proportional to theamount of sugar remaindafter different
time intervals. We nowhave,
Substituting the data inEquation,
for t = 1435 min
2.303 45.27
k = log = 5.442 10
5
1
1435 41.87
and for t = 11360 min
2.303 45.27
k = log = 5.311 10
5
1
11360 24.75
The values of k are fairly constant and so the reaction is of first order.
1
Ex.15 Bicyclohexane was found to undergo two parallel first order rearrangements. At 730 K, the first order
rate constant for the formation of cyclohexene was measurred as 1.26 10
4
s
1
, and for the formation
of methyl cyclopenten the rate constant was 3.8 10
5
s
1
. What is the percentage distribution of the
rearrangement products?
Sol.
k
1
Percentage of cyclohexene = 100
k + k
1 2
1.2610
4
= 100
1.2610
4
+ 3.810
5
= 77%.
percentage of methylcyclopenten =23%.
Ex.16 For the displacement reaction
[Co(NH ) Cl]
2+
+ H O [Co(NH ) (H O)]
3+
+ Cl

3 5 2 3 5 2
the rate constant is given by

11067 K
ln [k / (min
1
)] = + 31.33.
T
evaluate k, E andAfor the chemical reaction at 25 C.
Substituting T=298 Kin the given equation, we get,
k = 3.06 10
3
min
1
= 5.10 10
5
s
1
.
Sol.
36
Time (min) 0 1435 11360
Change in
rotation ()
34.50
(10.77)
= 45.27
(a)
31.10
(10.77)
= 41.87
(a x)
13.98
(10.77)
= 24.75
(a x)
10.77
(10.77)
= 0
Chemical Kinetics
Further, we have,
k =Ae
E/RT
E
or ln k = lnA
RT
Compairing this equation with the givenequation, we get,
lnA= 31.33
or 2.303 logA= 31.33 ; A= 4.04 10
13
min
1
= 6.73 10
11
s
1
E
and,
RT
=
11067 K
T
E = (11067 K) (8.314 J K
1
mol
1
)
= 92011 J/mole = 92.011 kJ/ mole.
The complexation of Fe
2+
withthe chelating agent dipyridyl has beenstudied kineticallyinbothforward
and reverse directions.
Fe
2+
+ 3 dipy Fe(dipy)
2+
Ex.17
3
Rate (forward) = (1.45 10
13
) [Fe
2+
][dipy]
3
and rate (reverse) = (1.22 10
4
) [Fe (dipy)
2+
].
3
Find the stabilityconstant for the complex.
At dynamic equilibrium,
rate of formation of complex =rate of its decomposition
(1.45 10
13
) [Fe
2+
][dipy]
3
= (1.22 10
4
) [Fe(dipy)
2+
]
Sol.
3
[Fe(dipy)
2+
] 1.4510
13
3
K = = = 1.19 10
17
.
2+ 3
1.2210
4
s
[Fe ][dipy]
Ex.18 The approach to the following equilibriumwas observed kinetically frombothdirection.
PtCl
2
+ H O Pt (H O)Cl

+ Cl

at 25 C.
4 2 2 3
d[PtCl
2
]
4
It was found that = (3.9 10
5
) [PtCl
2
)] (2.1 10
3
) [Pt(H O)Cl

] [Cl

]
dt
4 2 3
Calculate the equilibriumconstant for the complexation of the fourth Cl

byPt (II).
d[PtCl
2
]
4
Sol. At equilibrium, =0
dt
Hence, 3.9 10
5
[PtCl
2
]
= 2.1 10
3
[Pt(H O)Cl
2
][Cl

]
2 3 4
[PtCl
2
]
3
2.110 4
or K = = = 53.85.
[Pt(H O)Cl

][Cl

]
5
3.910 2 3
37
Chemical Kinetics
Ex.19 For the reaction
[Cr(H O) Cl ]
+
(aq)

k
1

[Cr(H O) Cl ]
2+
(aq)

k
2

[Cr(H O) ]
3+
(aq)
2 4 2 2 5 2 2 6
k = 1.78 10
3
s
1
and k = 5.8 10
5
s
1
for the initial concentration of [Cr(H O) Cl ]
+
is 0.0174
1 2 2 4 2
mole/litre at 0C. Calculate the value of t at which the concentration of [Cr(HO) Cl ]
2+
is maximum.
2 5 2
Sol. We have,
2.303(log k
1
log k
2
)
2.303(log1.7610
3
log5.810
5
)
t = = = 1990 seconds.
k k
3 5
1.7610 5.810 1 2
Ex.20 Fromthe following reaction scheme, write the rate lawfor the disappearance ofA, Band C.
(1) A+ B

k
1

C + D (2) C + D

k

A+ B (3) B + C

k
3

E + D
Sol. The reactant Ais removed inStep 1 and produced in Step 2
d[A]
= k [A] [B] k [C] [D]
1 2
dt
Similarly,
d[B]
= k [A] [B] + k [B] [C] k [C] [D]
1 3 2
dt
d[C]
and = k [C] [D] + k [B] [C] k [A] [B].
2 3 1
dt
38
Chemical Kinetics
EXERCISE- I
Q.1 Asolution ofAis mixed with an equal volume of a solution of Bcontaining the same number of moles,
and the reaction A+B=C occurs. At the end of 1h, A is 75 % reacted. How much of A will be left
unreacted at the end of 2 h if the reaction is (a) first order inAand zero order in B; (b) first order in both Aand
B; and (c) zero order in both Aand B ?
The reaction CH
3
CH
2
NO
2
+ OH

CH
3
CHNO
2
+ H
2
Oobeys the rate lawfor pseudo first
order kinetics inthe presence of a large excess of hydroxide ion. If 1%of nitro ethane undergoes reaction
inhalfa minute whenthereactant concentration is0.002 M, What is the pseudo first order rate constant?
The decomposition of a compound P, at temperature Taccording to the equation
2P
(g)
4Q
(g)
+ R
(g)
+ S
(l)
is the first order reaction. After 30 minutres fromthe start of decomposition in a closed vessel, the total
pressure developed is found to be 317 mmHg and after a long period of time the total pressure observed to be
617 mmHg. Calculate the total pressure of the vessel after 75 mintute, if volume of liquid S is supposed to
be negligible. Also calculate the time fraction t
7/8
.
Given : Vapour pressure of S (l) at temperature T= 32.5 mmHg.
A certain reactant B
n+
is getting converted to B
(n+4)+
in solution. The rate constant of this reaction is
measured by titrating a volume of the solution with a reducing reagent which only reacts with B
n+
and
B
(n+4)+
. In this process, it converts B
n+
toB
(n2)+
and B
(n+4)+
toB
(n1)+
. At t=0, the volume of the reagent
consumed is 25 ml and at t = 10 min, the volume used up is 32 ml. Calculate the rate constant of the
conversion of B
n+
to B
(n+4)+
assuming it to be a first order reaction.
Decomposition of H
2
O
2
is a first order reaction. Asolution of H
2
O
2
labelled as 20 volumes was left open.
Due to this, some H
2
O
2
decomposed. To determine the new volume strength after 6 hours, 10 mL of this
solutionwas diluted to 100mL. 10mLof this dilutedsolution was titrated against 25mLof 0.025M
KMnO
4
solution under acidic conditions. Calculate the rate constant for decomposition of H
2
O
2
.
Ametal slowly forms an oxide filmwhich completely protects the metal when the filmthickness is
3.956 thousand ths of an inch. If the film thickness is 1.281 thou. in 6 weeks, how much longer willit be
before it is 2.481 thou.? The rate of filmformation follows first order kinetics.
An opticallyactive compound Aupon acid catalysed hydrolysis yield two opticallyactive compound B and C
by pseudo first order kinetics. The observed rotation of the mixture after 20 min was 5 while after
completion of the reaction it was 20. If optical rotation per mole of A, B & C are 60,
40 & 80. Calculate half life of the reaction.
A vessel contains dimethyl ether at a pressure of 0.4 atm. Dimethyl ether decomposes as
CH
3
OCH
3
(g) CH
4
(g) +CO(g) + H
2
(g). The rate constant of decomposition is 4.7810
3
min
1
.
Calculate theratio of initial rate ofdiffusiontorateof diffusionafter 4.5hoursof initiationofdecomposition.
At roomtemperature (20C) orange juice gets spoilt in about 64 hours. In a referigerator at 3Cjuice can be
stored three times as long before it gets spoilt. Estimate (a) the activation energy of the reaction that causes
the spoiling of juice. (b) Howlong should it take for juice to get spoilt at 40C?
Afirst order reaction, AB, requires activation energyof 70 kJ mol
1
. When a 20%solution ofAwas kept at
25C for 20 minutes, 25% decomposition took place. What will be the percent decomposition in the same
time in a 30% solution maintianed at 40C? Assume that activation energy remains constant in this range of
temperature.
Two reations (i) A products (ii) B products, follow first order kinetics. The rate of the reaction (i) is
doubled when the temperature is raised from 300 K to 310K. The half life for this reaction at 310K is 30
minutes. At the same temperature Bdecomposes twice as fast as A. If the energyof activationfor the reaction
(ii) is half that of reaction (i), calculate the rate constant of the reaction (ii) at 300K.
Q.2
Q.3
Q.4
Q.5
Q.6
Q.7
Q.8
Q.9
Q.10
Q.11
39
Chemical Kinetics
Q.12 Acertain organic compoundAdecomposes by two parallel first order mechanism
If k
1
: k
2
= 1 : 9 and k
1
= 1.3 10
5
s
1
.
Calculate the concentration ratio of CtoA, if anexperiment is started withonlyAand allowed torun for
one hour.
The reaction Q.13
cisCr(en)
2
(OH)
+
2
transCr(en)
2
(OH)
+
2
is first order in both directions. At 25C the equilibriumconstant is 0.16 and the rate constant k
1
is
3.3 10
4
s
1
. In an experiment starting with the pure cis form, howlong would it take for half the
equilibrium amount of the trans isomer to be formed ?
Q.14 For a reversible firstorder reaction A B
k
1
= 10
2
s
1
and [B]
eq
/[A]
eq
= 4. If [A]
0
= 0.01 mole L
1
and [B]
0
= 0, what will be the concentration
of Bafter 30 s ?
For the systemA(g) lB(g), AHfor the forward reaction is 33 kJ/mol (Note : AH= AE in this case).
[B]
Showthat equilibriumconstant K=
[A]
= 5.572 10
5
at 300 K. If the activation energies E
f
&E
b
are
Q.15
in the ratio 20 : 31, calculate E
f
and E
b
at this temperature. Assume that the pre-exponential factor is the
same for the forward and backward reactions.
The complex [Co(NH
3
)
5
F]
2+
reacts with water according to the equation.
[Co(NH
3
)
5
F]
2+
+ H
2
O [Co(NH
3
)
5
(H
2
O)]
3+
+ F

The rate of the reaction = rate const. x[complex]


a
x[H
+
]
b
. The reaction is acid catalysed i.e. [H
+
] does
not change during the reaction. Thus rate = k
'
[Complex]
a
where k = k[H
+
]
b
, calculate a and b given
the following data at 25
0
C.
Q.16
[H
+
]M
0.01
0.02
[Complex]M
0.1
0.2
T
1/2
hr
1
0.5
T
3/4
hr
2
1
For the twoparallel reactionsA
k
1
B andA
k
2
C, showthat the activation energy E' for
the disappearance ofAis given in terms of activation energies E
1
and E
2
for the two paths by
Q.17
k
1
E
1
+ k
2
E
2
E' =
k + k
1 2
k
1 k
Q.18 For the mechanism A+ B C ; C
3
D
k
2
(a) Derive the rate lawusing the steady-state approximation to eliminate the concentration of C.
(b)Assuming that k
3
<< k
2
, express the pre-exponential factorAand E
a
for the apparent second-order
rate constant in terms ofA
1
, A
2
andA
3
and E
a1
, E
a2
and E
a3
for the three steps.
The reaction of formation of phosgene fromCOand Cl
2
is CO+ Cl
2
COCl
2
The proposed mechanism is
Q.19
(i)
(iii)
Cl
2
2Cl (fast equilibrium) (ii) Cl + CO COCl (fast equilibrium)
COCl + Cl
2

K
3

COCl
2
+ Cl (slow)

d[COCl
2
]
=K[CO][Cl
2
]
3/2
. Showthat the above mechanism leads to the following rate law
dt 1/ 2
k
k
| |
2
Where K = k
3
.
1
.
|
k
2 \
k 1
.
40
Chemical Kinetics
Q.20 For the following first order gaseous reaction
The initial pressure in a container of capacity Vlitres is 1 atm. Pressure at time t =10 sec is 1.4 atmand
after infinite time it becomes 1.5 atmosphere. Find the rate constant k
1
and k
2
for the appropriate
reactions.
Afirst order reaction takes 69.3 minutes for 50%completion. Howmuch time will be needed for 80%
completion.
The specific rate constant for a reaction increases bya factor 4, of the temperature is changed from
27Cto 47C. Find, the activation energyfor the reaction
Q.21
Q.22
Q.23 The reaction 2A+B+ C D+2 E is of first order withrespect toA, of second order with respect to
Band is of zero order withrespect to C
(i)
(ii)
Write down the rate lawfor the reaction
What will be the effect of doubling concentration ofA, Band C.
Q.24 Afirst order reaction is 50%completed in 30 minat 27Cand in10 minutes at 47C. Calculate the rate
constant at 27Cand the energy of activation of the reaction inkJ per mole.
The optical rotation of sucrose in 0.5 MHCl at 35Cat different time intervals are given below. Show
that the reaction follows first order kinetics.
Q.25
Time(min.) 0 10 20 30 40
+32.4 +28.8 +25.5 +22.4 +19.6 11.1 Rotation (degrees)
Q.26 T

of a reaction is halved as the initial concentration of the reaction is doubled. find out the order of the
reaction.
The rate constant of a reaction is 1.5 10
7
sec
1
at 50Cand 4.5 10
7
sec
1
at 100C. Evaluate the
Arrthenius parameters Aand Ea.
Asubstance reacts according to the lawof first order reaction the velocity constant of the reaction is
1.0 10
2
sec
1
. If initial canc. of the substance is 1.0 M.
Q.27
Q.28
(a)
(b)
Find out the initial rate
Find out the rate after 1 min.
What will be the initial rate of a reactionif itsrate constant is 10
3
min
1
andthe concentration of reactant
is 0.2 mol dm
3
. Howmuch of reactant will be converted into products in200 minutes.
Afirst order reaction is 20%complete in 10 minutes. Calculate
Q.29
Q.30
(a)
(b)
Specific rate constant of the reaction and
Time taken for the reaction to go to 75%completion
41
Chemical Kinetics
EXERCISE- II
Q.1 Ammonia andoxygen reacts at higher temperatures as
4NH
3
(g) + 5O
2
(g) 4NO(g) + 6H
2
O(g)
In an experiment, the concentrationof NOincreases by1.08 10
2
mol litre
1
in3 seconds. Calculate.
(i) rate of reaction. (ii) rate of disappearance of ammonia (iii) rate of formation of water
In the following reaction 2H
2
O
2
2H
2
O+ O
2
rate of formation of O
2
is 3.6 Mmin
1
.
(a) What is rate of formation of H
2
O?
(b) What is rate of disappearance of H
2
O
2
?
In a catalytic experiment involving the Haber process, N
2
+ 3H
2
2NH
3
, the rate of reaction was
measured as
Q.2
Q.3
A[NH
3
]
= 2 10
4
mol L
1
s
1
. Rate =
At
If there were no sides reactions, what was the rate of reaction expressed in terms of (a) N
2
, (b) H
2
?
The reaction 2A+ B+ CD+ E is found to be first order in Asecond order in Band zero order in C. (i)
Give the rate lawfor the reaction in the formof differential equation.
(ii) What is the effect inrate of increasing concentrations ofA, B, and Ctwo times?
At 27C it was observed during a reaction of hydrogenation that the pressure of hydrogen gas decreases
from2 atmosphere to 1.1 atmosphere in 75 minutes. Calculate the rate of reaction (in Msec
1
) andrate of
reaction in terms of pressure.
For the elementary reaction 2A+ B
2
2AB. Calculate how much the rate of reaction will change if
the volume of the vessel is reduced to one third of its original volume?
For the reaction 3BrO

BrO
3

+ 2Br

in analkaline aquesous solution, the value of the second


Q.4
Q.5
Q.6
Q.7

A[BrO ]
order (in BrO

) rate constant at 80Cinthe rate lawfor was found to be 0.056Lmol


1
s
1
.
At

A[BrO
3
]
, (b)
A[Br ]
? What is the rate of constant when the rate lawis written for (a)
Dinitropentaoxide decomposes as follows :
1
At At
Q.8
N
2
O
5
(g) 2NO
2
(g) +
2
O
2
(g)
Giventhat d [N
2
O
5
] / dt = k
1
[N
2
O
5
]
d [NO
2
] / dt = k
2
[N
2
O
5
]
d [O
2
] / dt = k
3
[N
2
O
5
]
What is the relation between k
1
, k
2
and k
3
?
Suppose that the Sun consists entirelyof hydrogen atomand releases the energybythe nuclear reaction, Q.9
4
1
H
4
He with 26 MeVof energy released. If the total output power of the Sunis assumed to
1 2
remain constant at 3.9 10
26
W, find the time it will take to burn all the hydrogen. Take the mass of the
Sun as 1.7 10
30
kg.
The reactionA(g) + 2B(g) C(g) + D(g) is an elementary process. In an experiment, the initial
partial pressure ofA&Bare P
A
= 0.6and P
B
=0.8 atm. Calculate the ratio of rate of reaction relative
to initial rate when P
C
becomes 0.2 atm.
In the following reaction, rate constant is 1.2 10
2
Ms
1
AB. What is concentration of Bafter
10 and 20 min., if we start with 10 MofA.
Q.10
Q.11
42
Chemical Kinetics
Q.12 For the following data for the zero order reactionAproducts. Calculate the value of k.
Time(min.)
0.0
1.0
2.0
[A]
0.10 M
0.09 M
0.08 M
The rate constant for a zero order reaction is 2 10
2
mol L
1
sec
1
, if the concentration of the reactant
after 25 sec is 0.25 M, calculate the initial concentration.
Adrop of solution (volume 0.10 ml) contains 6 10
6
mole of H
+
, if the rate constant of disappearance
of H
+
is 1 10
7
mole litre
1
sec
1
. Howlong would it take for H
+
in drop to disappear?
Acertain substanceAis mixedwithanequimolar quantityofsubstance B.At theend of an hourAis75%
reacted. Calculate the time whenAis 10%unreacted. (Given: order of reaction is zero)
Afirst order reaction is 75%completed in 72 min.. Howlong time will it take for
Q.13
Q.14
Q.15
Q.16
(i) 50%completion (ii) 87.5%completion
Q.17 Afirst order reaction is 20%complete in 10 min. calculate (i) the specific rate constant , (ii) the time
taken for the reactions to go to 75%completion.
Showthat in case of unimolecular reaction, the time required for 99.9%of the reaction to take place in ten
times that required for half of the reaction.
A first order reaction has a rate constant is 1.5 10
3
sec
1
. How long will 5.0 g of this reactant take to
reduce to 1.25 g.
A drug is known to be ineffective after it has decomposed 30%. The original concentration of a
sample was 500 units/ml. When analyzed 20 months later, the concentration was found to be
420 units/ml. Assuming that decomposition is of I order, what will be the expiry time of the drug?
A viral preparation was inactivated in a chemical bath. The inactivation process was found to be first
order in virus concentration. At the beginning of the experiment 2.0 % of the virus was found to be
inactivated per minute . Evaluate kfor inactivation process.
If a reaction A Products, the concentrations of reactant Aare C
0
, aC
0
, a
2
C
0
, a
3
C
0
, ............. after time
interval 0, t, 2t, 3t, ............. where a is a constant. Given 0 < a < 1. Show that the reaction is of first order.
Also calculate the relation in k, a and t.
The reaction SO
2
Cl
2
(g) SO
2
(g) + Cl
2
(g) is a first order gas reaction with k =2.2 10
5
sec
1
at
320C. What % of SO
2
Cl
2
is decomposed on heating this gas for 90 min.
Two substances A(t
1/2
= 5 mins) and B(t
1/2
=15 mins) followfirst order kinetics are taken in such a way that
initially[A]=4[B]. Calculate the time after which the concentration of boththe substance will be equal.
Q.18
Q.19
Q.20
Q.21
Q.22
Q.23
Q.24
Q.25 At 800 Cthe rate of reaction
2 NO + H
2
N
2
+ H
2
O
Changes withthe concentration of NOand H
2
are

1 d[NO]
1
[NO] in M [H
2
] in M
4 10
3
2 10
3
2 10
3
in M sec
2 dt
1.5 10
4
4.4 10
4
2.2 10
4
8.8 10
4
(i)
(ii) 1.5 10
4
(iii) 3.0 10
4
43
Chemical Kinetics
(a) What is the order of this reaction?
(b) What is the rate equation for the reaction?
(c) What is the rate when
[H
2
] = 1.5 10
3
M and [NO] = 1.1 10
3
M?
The data beloware for the reaction if NOand Cl
2
to formNOCl at 295 K Q.26
Initial Rate (Ms
1
)
1 10
3
3 10
3
9 10
3
Concentration of Cl
2
[M]
0.05
0.15
0.05
Concentration of NO
0.05
0.05
0.15
(a) What is the order w.r.t NOand Cl
2
in the reaction.
(b) Write the rate expression
(c) Calculate the rate constant
(d) Determine the reaction rate when concentration of Cl
2
and NOare 0.2M&0.4Mrespectively.
The catalytic decomposition of N
2
Oby gold at 900C and at an initial pressure of 200mm is 50%
complete in 53 minutes and 73%complete in100 minutes.
(i) What is the order of the reaction?
(ii) Calculate the velocityconstant.
(iii) Howmuchof N
2
O will decompose in100 min. at the same temperature but at initial pressureof 600
mm?
The pressure of a gas decomposing at the surface of a solid catalyst has been measured at different times and
the results are given below
Q.27
Q.28
t (sec)
Pr. (Pascal)
0
4 10
3
100
3.5 10
3
200
3 10
3
300
2.5 10
3
Determine the order of reaction, its rate constant.
The half life period of decomposition of a compound is 50 minutes. If the initial concentration is halved,
the half life period is reduced to 25 minutes. What is the order of reaction?
In this case we have
A B + C
Q.29
Q.30
Time
Total pressure of A+ B+C
Find k.
A B + C
Time
Total pressure of ( B+C)
Find k.
A B + C
Time
Volume of reagent
t
P
2

P
3
Q.31
t
P
2

P
3
Q.32
0 t
V
2
[Assuming n-factor ofA, B&Care same]
V
1
The reagent reacts withA, Band C. Find k.
44
Chemical Kinetics
Q.33 A 2B + 3C
Time
Volume of reagent
t
V
3
[Assuming n-factor ofA, B&Care same]
V
2
Reagent reacts with allA, Band C. Find k.
S G + F
Q.34
Time
Rotation of Glucose &Fructose
Find k.
t
r
t

3
The reactionAsH
3
(g) As(s) +
2
H
2
(g) was followed at constant volume at 310Cby measuring Q.35
the gas pressure at intervals Showfromthe following figures that reaction is of first order.
Time (inhrs)
Total pressure (in mm)
0
758
5
827
7.5
856
10
882
Q.36 The thermal decomposition of dimethyl ether as measured by finding the increase in pressure of the
(CH
3
)
2
O(g) CH
4
(g) + H
2
(g) + CO(g) reaction
at 500Cis as follows:
Time (sec.)

619
390
96
1195
250
3155
467
Pressure increase (mmHg)
the initial pressure of ether was 312 mmHg. Write the rate equation for this reaction and determine
the rate constant of reaction.
Fromthe following data showthat decomposition of H
2
O
2
in aqueous solution is first order.
Q.37
Time (in minutes)
Volume (in c.c. of KMnO
4
)
0
22.8
10
13.3
20
8.25
Q.38 The following data were obtained in experiment on inversion of cane sugar.

3.8
Time (minutes)
Angle of rotation (degree)
0
+13.1
60
+ 11.6
120
+ 10.2
180
+9.0
360
+5.87
Showthat the reaction is of first order.After what time wouldyou expect a zeroreading inpolarimeter?
At 100Cthe gaseous reaction A2B+ Cwas observed to be of first order. On starting with pure
Ait is found that at the end of 10 minutes the total pressure of systemis 176 mm. Hg and after a long time
270 mmHg. Fromthese data find (a) initial pressure ofA(b) the pressure ofAat the end of 10 minutes
(c) the specific rate of reaction and (d) the half life period of the reaction?
The decomposition of N
2
O
5
according to the equation 2 N
2
O
5
(g) 4 NO
2
(g) + O
2
(g) is a first
order reaction. After 30 min. from start of decomposition in a closed vessel the total pressure
developed is found to be 284.5 mm Hg. On complete decomposition, the total pressure is
584.5 mmHg. Calculate the rate constant of the reaction.
A definite volume of H
2
O
2
under going spontaneous decomposition required 22.8 c.c. of standard
permanganate solution for titration. After 10 and 20 minutes respectively the volumes of
permanganate required were 13.8 and 8.25 c.c.
(a) Find order of reaction. Howmay the result be explained?
(b) Calculate the time required for the decomposition to be half completed.
(c) Calculate the fraction of H
2
O
2
decomposed after 25 minutes.
Hydrogen peroxide solution was stored in a mild steel vessel. It was found, however, that the hydrogen
peroxide decomposed on the walls of the vessel (a first order reaction). An experiment with 100 ml of a
solution gave 10.31 ml oxygen (corrected to 1 atm &273 K) after 5.1 days. Find howlong the peroxide
can be stored before the loss of 20.00 ml oxygen occurs (per 100 ml solution) under similar storage
Q.39
Q.40
Q.41
Q.42
conditions. if complete decomposition of the sample to H
2
O
2
gave 46.34 ml oxygen.
45
Chemical Kinetics
The reaction given below, rate constant for disappearance ofAis 7.48 10
3
sec
1
. Calculate the time Q.43
required for the total pressure ina systemcontainingAat aninitial pressure of 0.1atmto rise to 0.145 atm
and also find the total pressure after 100 sec.
2A(g) 4B(g) + C(g)
The reaction A(aq) B(aq) +C(aq) is monitered bymeasuring optical rotation of reaction mixture
at different time interval. The speciesA, Band Care opticallyactive withspecific rotations 20, 30 and
40 respectively. Starting with pure Aif the value of optical rotation was found to be 2.5 after 6.93
minutes and optical rotation was 5 after infinite time. Find the rate constant for first order conversion
ofA into B and C.
Q.44
[x]
t
, calculate value of ratio,
[y] +[z]
at any given instant
t.
Q.45 For a reaction
[C]
k
1
= x hr
1
; k
1
: k
2
= 1 : 10. Calculate
[A]
after one hour fromthe start of the reaction. Q.46
Assuming onlyAwas present inthe beginning.
Howmuchtime would be required for the Bto reach maximum concentration for the reaction
L n2 k ln2
,
.
Q.47

k
1

k
2
A B C. Given k = =
1
2
4 2
For first order parallel reaction k
1
and k
2
are 8 and 2 min
1
respectively at
300 K. If the activation energies for the formation of Band Care respectively 20
and 28.314 kJ/mol respectively find the temperature at which Band Cwill
be obtained in molar ratio of 2 : 1.
[Given : ln 4 = 1.4 ]
In gaseous reactions important for understanding the upper atmosphere, H
2
Oand Oreact bimolecularly to
form two OH radicals. AH for this reaction is 72 kJ at 500 K and E
a
= 77 kJ mol
1
, then calculate E
a
for
the biolecular recombination of 2OHradicals to formH
2
O&Oat 500 K
The energyof activation of a first order reaction is 104.5 kJ mole
1
and pre exponential factor (A) is
5 10
13
sec
1
.At what temperature, will the reaction have a half life of 1 minute?
The specific rate constant for a reaction increases by a factor of 4, if the temperature is changed from
27Cto 47C. Find the activation energyfor the reaction.
The energyof activation and specific rate constant for a first order reaction at 25Care 100 kJ/ mole and
3.46 10
5
sec
1
respectively. Determine the temperature at which half life of the reaction is 2 hours.
Acatalyst lowers the activation energyfor a certain reactionfrom75 kJ to 25kJ mol
1
. What will be the
effect onthe rate of reaction at 25C, after things being equal.
Giventhat the temperature coefficient for the saponification of ethyl acetate byNaOHis 1.75. Calculate
activation energyfor the saponification of ethyl acetate.
At 380C, the halflife period for the first order decomposition of H O is 360 min.The energy of
Q.48
Q.49
Q.50
Q.51
Q.52
Q.53
Q.54
Q.55
2 2
activation of the reactionis 200 kJ mol
1
. Calculate the time required for 75%decomposition at 450
0
C.
46
Chemical Kinetics
Q.56 TheArrhenius equation for two first order equation
A B and C D is given by
k
1
= 10
12
k
2
= 10
11
e
81.28(kJ) / RT
e
43.10(kJ) / RT
At what temperature k
1
becomes equal to k
2
. The unit of activation energy is kJ/mol
Use: ln 10 = 2.3 and R = 8.3 J/K/mol
The reaction 2NO+ Br
2
2NOBr, is supposed to followthe following mechanism Q.57
(i) NO + Br
2
(ii) NOBr
2
+ NO
NOBr
2
slow

2NOBr
Suggest the rate lawexpression.
For the reaction 2H
2
+ 2NO N
2
+ 2H
2
O, the following mechanism has been suggested:
2NOl N
2
O
2
equilibrium constant K
1
(fast)
Q.58
k
N
2
O
2
+ H
2
N
2
O + H
2

k
3

N
2
O+ H
2
O (slow)
N
2
+ H
2
O (fast)
Establish the rate lawfor given reaction.
Reaction between NOand O
2
to formNO
2
is 2NO+ O
2
2NO
2
follows the following mechanism
Q.59
k
1
NO+ NO N
2
O
2
( in rapid equilibrium)
k1
N
2
O
2
+ O
2

k
2

2NO
2
(slow)
1 | d[ NO
2
] |
|
= K[NO]
2
[O ] Showthat the rate of reaction is given by
2
\
dt
.
2
Q.60 Deduce rate lawexpressions for the conversion of H
2
and I
2
to HI at 400Ccorresponding to each
of the following mechanisms:
(a) H
2
+ I
2
2HI (one step)
(b) I
2
2I
2I + H
2
2HI (slow)
(c) I
2
2I
I + H
2
IH
2
IH
2
+ I 2HI (slow)
(d) Can the observed rate lawexpressionrate = k[H
2
][I
2
] distinguish among these mechanisms?
(e) If it is knownthat ultraviolet light causes the reaction of H
2
and I
2
to proceed at 200Cwith the
same rate law expression, which of these mechanisms becomes most improbable?
47
Chemical Kinetics
EXERCISE- III
Q.1 The rate of a reaction is expressedin different ways as follows :
+
1 d[C]
=
1 d[D]
= +
1 d[A]
=
d[B]
2 dt 3 dt 4 dt dt
The reactionis:
(A) 4A+ B 2C + 3D
(C) A+ B C + D
(B) B + 3 D 4A+ 2 C
(D) B + D A+ C
For the reactionA+ B C; starting with different initial concentration ofAand B, initial rate of
reaction were determined graphically infour experiments.
Q.2
rate/(M sec )
Rate lawfor reaction fromabove data is
(A) r = k[A]
2
[B]
2
(B) r = k[A]
2
[B] (C) r = k[A] [B]
2
(D) r = k[A] [B]
Q.3 The rate lawfor a reaction between the substancesAand Bis given by
rate = k [A]
n
[B]
m
Ondoubling the concentration ofAand halving the concentration of B, the ratio of the newrate tothe
earlier rate of the reaction will be as
(A) 2
(nm)
(B) 1 / 2
(m+n)
(C) (m+ n) (D) (n m)
Q.4 For the reaction system 2NO(g) + O
2
(g) 2NO
2
(g) volume is suddenly reduced to half of its
value byincreasing the pressure onit. If the reaction is first order with respect to O
2
and second order
with respect to NO, the rate of reaction will
(A) increase to four times of its initial value
(C) diminishto one-eight of its initial value
(B) diminishto one-fourth of its initial value
(D) increase to eight times of its initial value
Q.5 In a first order reaction, the concentration of the reactant, decreases from0.8 to 0.4 Min 15 minutes.
The time taken for the concentration to change from0.1 Mto 0.025 Mis
(A) 30 minutes (B) 15minutes (C) 7.5minutes (D) 60minutes
The rate equation for the reaction 2A+BCis found to be : rate =k[A] [B]. The correct statement
in relation to this reaction is
(A) unit of k must be s
1
(B) t
1/2
is a constant
(C) rate of formation of C is twice the rate of disappearance ofA
(D) value of k is independent of the initial concentrations ofAand B.
t
1/4
can be taken as the time taken for the concentration of a reactant to drop to 3/4 of its value. If the
rate constant for a first order reaction is k, the t
1/4
can be written as [ln2 = 0.695, ln = 1.1]
Q.6
Q.7
(A) 0.69 / k (B) 0.75 / k (C) 0.10 / k (D) 0.29 / k
Q.8 Areaction was found to be second order with respect to the concentration of carbon monoxide. If the
concentrationofcarbon monoxide is doubled, witheverything else kept the same, the rate of reaction will
(A) Double (B) remain unchanged (C) Triple (D) Increase by a factor of 4
48
S.No. [A]
0
/M (Initial conc.) [B]
0
/M (Initial conc.)
1
1 1.6 10
3
5 10
2
10
3
2 3.2 10
3
5 10
2
4 10
3
3 1.6 10
3
10
1
2 10
3
4 3.2 10
3
10
1
8 10
3
Chemical Kinetics
2ABare 180 kJ mol
1
Q.9
The energies of activation for forward and reverse reactions for A
2
+ B
2
and 200 kJ mol
1
respectively. The presence of catalyst lowers the activation energies of both (forward
and reverse) reactions by 100 kJ mol
1
. The magnitude of enthalpy change
(A
2
+ B
2
2AB) in the presence of catalyst will be (in kJ mol
1
).
of the reaction
(A) 300 (B) 120 (C) 20 (D) 20
Which graphrepresents zeroorder reaction [A(g) B(g)] : Q.10
d[B]
(A)
[B]
dt
(B)
t
t
t
1/2
t
3/4
(C) (D)
[A]
0
[A]
0
For a hypothetical reaction,
A+ 3B P
Q.11
AH = 2 x kJ/mole ofA
AH = + x kJ/mole of M & M 2Q + R
If these reactions are carried simultaneouslyina reactor suchthat temperature is not changing. Ifrate of
disappearance of Bis y Msec
1
then rate of formation (in Msec
1
) of Qis :
2
(A)
3
y
3
(B)
2
y
4
(C)
3
y
3
(D) y
4
Q.12 Gaseous reactionAB+ Cfollows first order kinetics. Concentration ofAchanges from1 Mto
0.25 Min 138.6 minutes. Find the rate of reaction when concentration ofAis 0.1 M.
(A) 210
3
Mmin
1
(B) 10
3
Mmin
1
(C) 10
4
M min
1
(D) 5 10
4
M min
1
The initial rate of zero order reaction of the gaseous reactionA(g) 2B(g) is 10
2
Mmin
1
. If the
initial concentration ofAis 0.1 M, what would be the concentration of Bafter 60 sec.?
Q.13
(A) 0.09 M (B) 0.01 M (C) 0.02 M (D) 0.002 M
Q.14 Consider the following first order competing reactions:
Y
k
2
C + D
X
k
1
A+ B
and
if 50%of the reaction of Xwas completed when96%of the reaction ofYwas completed, the ratio of
their rate constants (k
2
/k
1
) is
(A) 4.06 (B) 0.215 (C) 1.1 (D) 4.65
Q.15 Consider the reaction A B, graph between half life (t
1/2
) and initial concentration (a) of the reactant
is
t
1/2
a

d[A]
Hence graph between and time will be
dt
49
Chemical Kinetics

d[A]

d[A]
(A) (B)
dt dt
t

d[A]

d[A]
(C) (D)
dt dt
t t
Q.16 At certaintemperature, the half life period for the thermal decomposition of a gaseous substance depends
on the initial partial pressure of the substance as follows
P(mmHg)
t
1 2
(inmin.)
500
235
250
950
Find the order of reaction [Given log (23.5) = 1.37 ; log (95) = 1.97; log 2 = 0.30]
(A) 1 (B) 2 (C) 2.5 (D) 3
Q.17 The reactions of higher order are rare because
(A) manybodycollisions involve veryhigh activation energy
(B) manybodycollisions have a verylowprobability
(C) manybody collisions are not energetically favoured.
(D) manybody collisions can take place only in the gaseous phase.
Consider the reaction :
A B + C
Initial concentration ofAis 1 M. 20 minutes time is required for completion of 20 %reaction.
d[B]
Q.18
If = k[A], then half life (t
1/2
) is
dt
(A) 55.44 min. (B) 50 min (C) 62.13 min (D) None of these
Q.19 If decompositionreactionA(g) B(g) follows first order kinetics then the graphof rate of formation
(R) of Bagainst time t will be
(A) (B) (C) (D)
Q.20 For the first order decomposition of SO
2
Cl
2
(g),
SO
2
Cl
2
(g) SO
2
(g) + Cl
2
(g)
a graph of log (a
0
x) vs t is shown in figure. What is the rate constant (sec
1
)?
Time (min)
2 4 6 8
|
10
|
(0,0)
| | |
-1
-2
-3
(B) 4.6 10
1
(C) 7.7 10
3
(D) 1.15 10
2
(A) 0.2
50
l
o
g
(
a
0

x
)
Chemical Kinetics
The rate constant for the forward reactionA(g) l 2B(g) is 1.5 10
3
s
1
at 100 K. If 10
5
moles ofA
and 100 moles of Bare present in a 10litre vessel at equilibriumthenrate constant for the backward
reaction at this temperature is
Q.21
(A) 1.50 10
4
Lmol
1
s
1
(C) 1.5 10
10
Lmol
1
s
1
(B) 1.5 10
11
Lmol
1
s
1
(D) 1.5 10
11
Lmol
1
s
1
+
1 1
ReactionA+BC+Dfollows following rate law: rate = k[A]
2
[B]
2
. Starting withinitial conc.
of 1 MofAand Beach, what is the time taken for concentration ofAof become 0.25 M.
Given : k = 2.303 10
3
sec
1
.
Q.22
(A) 300 sec. (B) 600 sec. (C) 900 sec. (D) 1200 sec.
The reactionA(g) B(g) + 2C (g) is a first order reaction with rate constant 3465 10
6
s
1
.
Starting with0.1 mole ofAin2 litre vessel, find the concentration ofAafter 200 sec., whenthereaction
is allowed to take place at constant pressure and temperature.
Q.23
(A) 0.05 M (B) 0.025 M (C) 0.0125 M (D) None of these
Q.24 Decomposition of H
2
O
2
is a first order reaction. Asolution of H
2
O
2
labelled as "16.8 V" was left open.
Due to this, some H
2
O
2
decomposed. To det ermine the new volume strengt h after
2.303 hours, 20 mLof this solution was diluted to 100 mL. 25 mLof this diluted solution was titrated
against 37.5 mLof 0.02 MKMnO
4
solution under acidic conditions [Given : STPis 1atmand 273 K]
The rate constant (in hr
1
) for decomposition of H
2
O
2
is :
(A) 0.15 (B) 0.30 (C) 0.60 (D) 1.3
The variation of concentration ofAwith time in two experiments starting with two different initial
concentration ofAis given in the following graph. The reaction is represented asA(aq) B(aq).
What is the rate of reaction (M/min) whenconcentration ofAin aqueous solutionwas 1.8 M?
1.5
1.2
1
0.8
0.6
Experiment-1
Experiment-2
5 10 15 20
time(min.)
(B) 0.036 Mmin
1
(D) 1 Mmin
1
(A) 0.08 Mmin
1
(C) 0.13 Mmin
1
Q.26
In respect of the equation k =Aexp ( E
a
/ RT), which one of the following statements is correct?
(A) Ris Rydberg's constant
(C)Ais adsorption factor
(B) kis equilibriumconstant
(D) E
a
is the energyof activation
Q.27 Rate of a reaction can be expressedbyArrhenius equation as :
k =Ae
E/RT
In this equation, Erepresents
(A) The fractionof molecules with energygreater than the activation energyof the reaction
(B) The energyabove whichall the colliding molecules will react
(C) The energybelowwhich colliding molecules will not react
(D) The total energyof the reacting molecules at a temperature, T
51
C
o
n
c
e
n
t
r
a
t
i
o
n
(
M
)

Chemical Kinetics
Q.28 The rate constant, the activationenergyandtheArrhenius parameter (A) of achemical reaction at 25Care
3.0 10
4
s
1
, 104.4 kJ mol
1
and 6.0 10
14
s
1
respectively. The value of the rate constant at Tis
(A) 2.0 10
18
s
1
(B) 6.0 10
14
s
1
(C) infinity (D) 3.6 10
30
s
1
Q.29 Afirst order reaction is 50% completed in 20 minutes at 27Cand in 5 min at 47C. The energy of
activation of the reaction is
(A) 43.85kJ/mol (B) 55.14 kJ/mol (C) 11.97 kJ/mol (D) 6.65 kJ/mol
For the first order reactionAB+C, carried out at 27 Cif 3.8 10
16
%of the reactant molecules
exists in the activated state, the E
a
(activation energy) of the reaction is [log3.8 = 0.58]
Q.30
(A) 12 kJ/mole (B) 831.4 kJ/mole (C) 100 kJ/mole (D) 88.57 kJ/mole
Q.31 In a reaction carried out at 400 K, 0.0001% of the total number of collisions are effective. The energyof
activation of the reaction is
(A) zero (B) 7.37 k cal/mol (C) 9.212 k cal/mol (D) 11.05 k cal/mol
Q.32 The following mechanismhas been proposedfor the exothermic catalyzed complex reaction.

k
1
AB + I
k
2
P+A A+ B I AB
If k
1
is much smaller than k
2
. The most suitable qualitative plot of potential energy (P.E.) versus reaction
coordinate for the above reaction.
Q.33 The following mechanism has been proposed for the reaction of NOwith Br
2
to form NOBr :
NO(g) + Br
2
(g) NOBr
2
(g)
NOBr
2
(g) + NO(g) 2NOBr(g)
If the second step is the rate determining step, the order of the reaction with respect to NO(g) is
(A) 2 (B) 1 (C) 0 (D) 3
Q.34 Choose the correct set of identifications.
(1)
AE for
E + S
E
a
for
E + S
E
a
for
(2)
E
a
for
ES EP
AEfor
E + S ES
E
a
for
EP E + P
E
a
for
ES EP
AE
overall
for S P
(3)
AE
overall
for S P
E
a
for
ES EP
AE
overall
for S P
E
a
for
EP E + P
AE for
l
EP E + P
(4)
E
a
for
EP E + P
AE
over all
for S P
AEfor
EP E + P
AE
overall
for S P
E
a
for
EP E + P
(A)
ES
(B)
ES
(C)
ES
E
a
for
EP
(D)
E + S
AE for
E + S
ES
(E)
ES
52
Chemical Kinetics
SO
3
gas is entering the environment at a constant rate of 6.93 10
6
gm/L/day due to the emission of Q.35
polluting gases fromthermal power plant, but at the same time it is decomposing &following first order
kinetics withhalf life of 100 days.
Based onabove information select the true statement(s).
(A) Concentration of SO
3
inKota is 1.25 10
5
M(Assume SO
3
present in air reaches steady state)
(B) If 10
3
Lof air is passed through 1 Lpure water (assuming all SO
3
to be dissolved in it) &resulting
solution is titrated against 1 NNaOHsolution, 15ml is required to reach end point.
(C)Anindustryis manufacturing H
2
SO
4
at the rate of 980 kg per daywith the useof SO
3
inair it should
use 8 10
5
Litre air /day.
(D) If SO
3
emission is stopped then after 1000 days its concentrations will reduce to ~

1.2 10
8
M.

d[A]
For the reactionAB, the rate lawexpression is
1/2
Q.36 = k [A] . If initial concentration of [A] is
d t
[A]
0
, then
2
1/ 2 1/ 2
(A
0
A )
(A) The integerated rate expression is k =
t
A (B) The graph of Vs t will be
K
t
1/ 2
= (C) The half life period
1/ 2
2[A]
0
t
3/ 4
= (D) The time taken for 75%completion of reaction
Consider the reaction,
k
Q.37
B
A
C
A, BandCall are opticallyactive compound . If optical rotation per unit concentration ofA, Band Care
60, 72, 42 and initial concentration ofAis 2 Mthen select write statement(s).
(A) Solution will be opticallyactive and dextro after verylong time
(B) Solutionwill be optically active and levo after verylong time
(C) Half life of reaction is 15 min
(D)After 75%conversion ofAinto Band Cangle of rotation of solution will be 36.
Select incorrect statement(s):
(A) Unit of pre-exponential factor (A) for second order reaction is mol L
1
s
1
.
(B)Azero order reaction must be a complex reaction.
(C) Molecularity is defined onlyfor RDSina complex reaction.
(D) Decayconstant () of radioactive substance is affected bytemperature.
Q.38
In a consecutive reactionsystemA
E
1
B
E
2
CwhenE is muchgreater thanE , the yield of B
Q.39
1 2
increase with
(A) increase in temperature
(C) increase ininitial concentration ofA
(B) decreases intemperature
(D) decrease ininitial concentration ofA
53
[A]
0

Chemical Kinetics
Q.40 Which of the following is/are correct statement?
(A) Stoichiometryof a reaction tells about the order of the elementary reactions.
(B) For a zero order reaction, rate and the rate constant are identical.
(C)Azero order reaction is controlled by factors other thanconcentration of reactants.
(D)Azero order reaction is always elementary reaction.
Which of the following statement isincorrect?
(A) The order of reaction is the sumof powers of all the concentration terms in the rate equation.
(B) The order of reaction with respect to one reactant is the ratio of the change of logarithm of the rate
of the reaction to the change in the logarithmof the concentration of the particular reactant, keeping the
concentrations of all other reactants constant.
(C) Orders of reactions can not be fractional.
(D) The order of a reaction can onlybe determined fromthe stoichiometric equation for the reaction.
Q.41
Statement Type Question :
Q.42 Statement-1 :Afractional order reaction must be a complex reaction.
Statement-2 : Fractional order of RDSequals to overall order of a complex reaction.
(A) Statement-1 is true, statement-2 is true and statement-2 is correct explanationfor statement-1.
(B) Statement-1is true, statement-2 istrue andstatement-2 is NOTthe correct explanationfor statement-1.
(C) Statement-1 is true, statement-2 is false.
(D) Statement-1 is false, statement-2 is true.
Statement-1 : The time of completion of reactions of typeA product (order <1) may be
determined.
Statement-2 : Reactions with order > 1 are either too slowor too fast and hence the time of
completion cannot be determined.
(A) Statement-1 is true, statement-2 is true and statement-2 is correct explanationfor statement-1.
(B) Statement-1is true, statement-2 istrue andstatement-2 is NOTthe correct explanationfor statement-1.
(C) Statement-1 is true, statement-2 is false.
(D) Statement-1 is false, statement-2 is true.
Statement-1 : Temperature coefficient of an one step reactionmay be negative.
Statement-2 : The rate of reaction having negative order withrespect to a reactant decreases
with the increase in concentration of the reactant.
(A) Statement-1 is true, statement-2 is true and statement-2 is correct explanationfor statement-1.
(B) Statement-1is true, statement-2 istrue andstatement-2 is NOTthe correct explanationfor statement-1.
(C) Statement-1 is true, statement-2 is false.
(D) Statement-1 is false, statement-2 is true.
Statement-1 : The overall rate of a reversible reaction may decrease withthe increase in
temperature.
Statement-2 : When the activation energy of forward reaction is less thanthat of backward
reaction, then the increase in the rate of backward reaction is more thanthat of
forward reaction onincreasing the temperature.
(A) Statement-1 is true, statement-2 is true and statement-2 is correct explanationfor statement-1.
(B) Statement-1is true, statement-2 istrue andstatement-2 is NOTthe correct explanationfor statement-1.
(C) Statement-1 is true, statement-2 is false.
(D) Statement-1 is false, statement-2 is true.
Q.43
Q.44
Q.45
54
Chemical Kinetics
Q.46 Statement-1 : In a reversible endothermic reaction, E of forward reactionis higher than that
act
of backward reaction
Statement-2 : The threshold energy of forward reaction is more thanthat of backward reaction
(A) Statement-1 is true, statement-2 is true and statement-2 is correct explanationfor statement-1.
(B) Statement-1is true, statement-2 istrue andstatement-2 is NOTthe correct explanationfor statement-1.
(C) Statement-1 is true, statement-2 is false.
(D) Statement-1 is false, statement-2 is true.
Statement-1 : Acatalyst provides analternative path to the reaction inwhichconversion of
reactants into products takes place quickly
Statement-2 : The catalyst forms anactivated complexof lower potential energy, with the
reactants by which more number of molecules are able to cross the barrier per
unit of time.
(A) Statement-1 is true, statement-2 is true and statement-2 is correct explanationfor statement-1.
(B) Statement-1is true, statement-2 istrue andstatement-2 is NOTthe correct explanationfor statement-1.
(C) Statement-1 is true, statement-2 is false.
(D) Statement-1 is false, statement-2 is true.
Q.47
Paragraph 1 :
Oxidation of metals is generally a slow electrochemical reaction involving many steps. These steps
involve electron transfer reactions. Aparticular type of oxidation involve overall first order kinetics with
respect to fraction of unoxidised metal (1 f ) surface thickness relative to maximum thickness (T) of
oxidised surface, whenmetal surface is exposed to air for considerable period of time
df
Rate law : = k(1 f ), where f = x/T,
dt
x = thickness of oxide filmat time 't'
& T = thickness of oxide film at t =
Agraph of ln(1 f ) vs t is shown inthe adjacent figure.
Q.48 The time taken for thickness to grow50%of 'T' is
(A) 23.1 hrs (B) 46.2 hrs (C) 100 hrs (D) 92.4 hrs
Q.49 The exponential variation of 'f' witht(hrs) is given by
(A)
[1 e
3t / 200
]
Paragraph 2 :
(B)
e
3t / 200
1
(C)
e
3t / 200
(D)
e
3t / 200
k
1
=
1
For a hypothetical elementary reaction where
k
2
2
Initially only2 moles ofAare present.
Q.50 The total number of moles of A, B&Cat the end of 50%reaction are
(A) 2 (B) 3 (C) 4 (D) 5
Q.51 Number of moles of Bare
(A) 2 (B) 1 (C) 0.666 (D) 0.333
55
Chemical Kinetics
Paragraph 3 :
Areaction is said to be first order if it's rate is proportional to the concentration of reactant. Let us
consider a reaction
A(g)
a
a x
B(g)
0
x
+ C(g)
0
x
At t = 0
At time t
dx
The rate of reaction is given by the expression = k(a x) and integrated rate equation for a
given
dt
a 1 | |
reaction is represented as k = ln
a x
| where a = initial concentration and (a x) = concentration
t \ .
ofAafter time t.
Thermal decomposition of compound Xis a first order reaction. If 75%of Xis decomposed in 100 Q.52
min. Howlong will it take for 90%of the compound to decompose? Given : log 2 = 0.30
(D) 156.66min (A) 190 min (B) 176.66min (C) 166.66 min
Consider a reaction A(g) 3B(g) + 2C(g) with rate constant 1.386 10
2
min
1
. Starting with
2 moles ofAin 12.5 litre vessel initially, if reaction is allowed to takes place at constant pressure &at
298Kthen findthe concentration of Bafter 100 min.
Q.53
(A) 0.04 M
Paragraph 4 :
(B) 0.36 M (C) 0.09 M (D) None of these
The gaseous reaction : n
1
A(g) n
2
B(g) is first order with respect toA. The true rate constant of
reaction is k. The reaction is studied at a constant pressure and temperature. Initially, the moles ofA
were 'a' and no B were present.
Howmany moles ofAare present at time, t?
Q.54
(A) a e
kt
(B)
a e
n
1
kt
a e
n
2
kt
(D)
a (1 e
n
1
kt
)
(C)
Q.55
If the initial volume of system were v
0
, then the volume of system after time, t, will be
n
1
v
0
n
2
v
0
(A) (B)
n n
2 1

n
(
n
(
| | | n | n
2
| e
n
1
kt
(
1| e
n
1
kt
(
2 2 2
+ 1 v v

(C) (D) 0 0

n
1 \
n
1 .
n
1 \
n
1 .

Q.56 What will be the concentration of Aat time t, if n
1
= 1 and n
2
= 2?
e
kt
e
kt
| | | |
]
|
]
|
[A [A
(A) [A
0
] e
kt
Paragraph 5 :
kt
(B) (C) (D) [A
0
] (12 e )

2 e
kt |
1 e
kt |
0 0
\ . \ .
For the given sequential reaction
A
k
1
B
k
2
C
the concentration ofA, B&Cat anytime 't' is given by
k
1
[A]
0
|e
k
1
t
e
k
2
t
|
k t
[A]
t
= [A]
0
e ; [B]
t
=
1
(k k )
2 1
[C]
t
= [A
0
] ( [A]
t
+ [B]
t
)
56
Chemical Kinetics
Q.57 The time at whichconcentration of Bis maximumis
k
1
1 k
1
1 k
1
k
2
ln ln
(A) (B) (C) (D)
k k k
2
k
1
k
2
k
1
k
2
k
2
k k
2 1 2 1
Select the correct option if k
1
= 1000 s
1
and k
2
= 20 s
1
. Q.58
Match the Column:
Q.59 For the reaction of typeA(g) 2B(g)
Column-I contains four entries andcolumn-II contains four entries. Entryofcolumn-I areto bematched
withonly one entryof column-II
ColumnI ColumnII
d[B] d[A]
(A) vs for first order (P)
dt dt
(B) [A] vs t for first order (Q)
(C) [B] vs t for first order (R)
(D) [A] vs t for zero order (S)
57
Chemical Kinetics
Q.60 Column-I and column-II. Entryof column-I are to be matched withone or more thanone entries of
column-II and vice versa.
ColumnI
(Graphs reactionA Products)
ColumnII
(Co-ordinates)
(A) (P) ln [A] (y-axis), t (x-axis) (order = 1)
(B) (Q) t
1/2
(y-axis), [A
0
] (x-axis) (order = 1)
(C) (R) r (y-axis), t (x-axis) (order > 0)
(D) (S) r (y-axis), t (x-axis) (order = 0)
(T) t
1/2
(y-axis), [A
0
] (x-axis) (order > 1)
1
(U) (y-axis), t (x-axis) (order = 2)
[A]
(V) r (y-axis), [A] (x-axis) (order = 1)
58
Chemical Kinetics
ANSWERKEY
EXERCISE -I
2.
4.
6.
8.
10.
12.
14.
16.
2 10
2
min
1
0.0207 min
1
15.13 week
0.26 : 1
%decomposition= 67.21%
0.537
0.0025 m
a = b = 1
1.
3.
5.
7.
9.
11.
13.
15.
(a) 6.25 ; (b) 14.3 ; (c) 0%
P = 379.55 mmHg, t
7/8
= 399.96min
t
k = 0.022 hr
1
20 min
(a) 43.46kJmol-1, (b) 20.47 hour
k = 0.0327 min
1
4.83 mins
E
f
= 6 10
4
J; E = 9.3 10 J
4
b
d(D) k
1
k
3
(A)(B) A
1
A
3
=
18. (a) ; (b) E
a
= E
a1
+ E
a3
E
a2
. A=
dt k + k A
2 3 2
20.
22.
24.
25.
27.
0.0805
55.33 kJ mole
1
21. 161 minutes
23. rate = K[A] [13]
2
rate will became 8 times.
K
27
= 3.85 10
4
sec
1
K
47
= 11.55 10
4
sec
1
,
E = 43.78 kJ/mol
It is first order kinetics with k = 8.64 10
3
Ea = 2.2 10
4
A= 5.42 10
10
(a) = 1 10
2
mol l
1
s
1
(b) = 5.495 10
3
mol
1
l
1
s
1
Rate = 2 10
4
mol dm
3
min
1
x = 18.12%
K= 0.02231 min
1
t = 62.07 min
EXERCISE-II
1 d[NO]
26. 2
28.
29.
30.
4 1 1 4 1 1 4 1 1
1. (i) r =
4
= 9 10 mol litre sec , (ii) 36 10 mol litre sec , (iii) 5410 mol litre sec
dt
(i) 7.2 mol litre
1
min
1
, (ii) 7.2 mol litre
1
min
1
(a) 1 10
4
mol L
1
s
1
, (b) 3 10
4
mol L
1
s
1
dx
2.
3.
(i) = k[A][B]
2
, (ii) rate increases by 8 times 4.
5.
7.
9.
11.
13.
15.
17.
20.
dt
8.12 10
6
Ms
1
, 0.012 atm min
1
(a) 0.019 mol L
1
s
1
, (b) 0.037 mol L
1
s
1
6.
8.
10.
12.
14.
16.
19.
21.
rate increase by 27 times
2k
1
= k
2
= 4k
3
1/6
K= 0.01 M min
1
6 10
9
sec
(i) 36 min., (ii) 108 min.
924.362 sec
3.3 10
4
s
1
8 310
18
sec
(i) 7.2 M, (ii) Think
0.75 M
1.2 hr
(i) 0.0223 min
1
, (ii) 62.17 min
expirytime =41months
59
Chemical Kinetics
2.303
log
1
22.
24.
25.
26.
k = 23. 11.2%
t a
15min
(a) Third order, (b) r = k[NO]
2
[H
2
], (c) 8.85 10
3
Msec
1
.
r = K[NO]
2
[Cl
2
], (c) K = 8 L
2
mol
2
s
1
, (a) order w.r.t NO = 2 and w.r.t Cl
2
= 1, (b)
(d) rate = 0.256 mole L
1
s
1
(i) first order (ii) k = 1.308 10
2
min
1
(iii) 73%
27. 28. (i) Zero order, (ii) K=5 Pa/s
l
ln
P
3
29. Zero order 30. k =
t 2(P
3
P
2
)
V
1 l
ln
P
3
l
k =
ln
31. k =
t
32.
(P
3
P
2
)
4V
3
t (2V
1
V
2
)
l
ln
l
k =
ln
r

33.
35.
37.
39.
40.
41.
k =
t
34.
36.
38.
5(V
3
V
2
) t (r

r
t
)
(i) r = K[(CH
3
)
2
O], 0.000428 sec
1
966 min
First order
First order
(a) 90 mm, (b) 47mm, (c) 6.49 10
2
per minutes, (d) 10.677min.
k
1
= 2.605 10
3
min
1
(a) first order, (b) 13.75 minutes, (c) 0.716
0.180 atm, 47.69 sec
1
42.
44.
11.45 days
0.1 min
1
[C]
10
11x
45. 46. = (e 1)
(K
1
+K
2
)
[A]
t 1
11 e
47.
49.
51.
53.
55.
57.
60.
t =4 min
5 kJ mol
1
55.33 kJ mole
1
rate of reaction increases 5.81 10
8
times
t =20.4 minutes
r = K' [NO]
2
[Br
2
]
(d) No, (e) mechanism (a) is incorrect
EXERCISE
48.
50.
52.
54.
56.
58.
0379.75 K
349.1 k
306 k
10.757 k cal mol
1
2000 K
r = K[NO]
2
[H
2
], where K= k
2
K
1
-III
5. A
12. B
19. C
26. D
33. A
40. A,B,C
47. A
54. B
1. B
8. D
15. C
22. B
29. B
36. A,B,D
43. C
50. B
57. C
2. B
9. C
16. D
23. B
30. C
37. A,D
44. D
51. C
58. C
3. A
10. D
17. B
24. C
31. D
38. A,C,D
45. A
52. C
4. D
11. C
18. C
25. A
32. A
39. A,C
46. C
53. C
6. D
13. C
20. C
27. C
34. B
41. C,D
48. B
55. D
7. D
14. D
21. D
28. B
35. A,D
42. C
49. A
56. B
59. (A) S, (B) R, (C) P, (D) Q 60. (A) P (B) Q,S (C) R,T(D) V
60

You might also like